Cardiac Arrest

Brief HPI:

An overhead page alerts you to an arriving patient with cardiac arrest. An approximately 35-year-old male was running away from police officers and collapsed after being shot with a stun gun. The patient was found to be pulseless, CPR was started by police officers and the patient is en route.

An Algorithm for the Evaluation and Management of Cardiac Arrest with Ultrasonography

An Algorithm for the Evaluation and Management of Cardiac Arrest with Ultrasonography

Causes of Cardiac (and non-cardiac) Arrest

Sudden cardiac arrest (SCA) leading to sudden cardiac death (SCD) if not successfully resuscitated, refers to the unexpected collapse of circulatory function. Available epidemiologic data for in-hospital and out-of-hospital cardiac arrest (OHCA) point appropriately to cardiac processes as the most common cause, though extra-cardiac processes (most frequently respiratory), comprise up to 40% of cases1-3.

Identifying the underlying cause is critical as several reversible precipitants require rapid identification. However, the usual diagnostic techniques may be challenging, limited or absent – including patient history, detailed examination, and diagnostic studies.

The initial rhythm detected upon evaluation is most suggestive of the etiologic precipitant. Pulseless ventricular tachycardia (pVT) or ventricular fibrillation (VF) is suggestive of a cardiac process – most commonly an acute coronary syndrome although heart failure or other structural and non-structural heart defects associated with dysrhythmias may be at fault4.

Pulseless electrical activity (PEA) presents a broader differential diagnosis as it essentially represents severe shock. The most common extra-cardiac cause is hypoxia – commonly secondary to pulmonary processes including small and large airway obstruction (bronchospasm, aspiration, foreign body, edema). Other causes include substance intoxication, medication adverse effect5,6, or electrolyte disturbances7. Finally, any precipitant of shock may ultimately lead to PEA, including hypovolemia/hemorrhage, obstruction (massive pulmonary embolus8, tamponade, tension pneumothorax), and distribution (sepsis).

Asystole is the absence of even disorganized electrical discharge and is the terminal degeneration of any of the previously-mentioned rhythms if left untreated.

Management of Cardiac Arrest

Optimizing survival outcomes in patients with cardiac arrest is dependent on early resuscitation with the prioritization of interventions demonstrated to have survival benefit. When advanced notice is available, prepare the resuscitation area including airway equipment (with adjuncts to assist ventilation and waveform capnography devices). Adopt the leadership position and assign roles for chest compressions, airway support, application of monitor/defibrillator, and establishment of peripheral access.

High-quality chest compressions with minimal interruptions are the foundation of successful resuscitation – and guideline changes prioritizing compressions have demonstrated detectable improvements in rates of successful resuscitation9,10. Measurement of quantitative end-tidal capnography can guide adequacy of chest compressions11,12 and an abrupt increase may signal restoration of circulation without necessitating interruptions of chest compressions13,14. Sustained, low measures of end-tidal CO2 despite appropriate resuscitation may signal futility and (alongside other factors) guides termination of resuscitation11,12.

The next critical step in restoring circulation is prompt defibrillation of eligible rhythms (pVT/VF) when detected. The immediate delivery of 200J (uptitrated to the device maximum for subsequent shocks) of biphasic energy and restoration of a perfusing rhythm is one of few interventions with clear benefits. For pVT/VF that persists despite multiple countershocks (more than three), the addition of an intravenous antiarrhythmic appears to improve survival to hospital admission. The ARREST trial was a randomized controlled study comparing amiodarone to its diluent as placebo for OHCA with refractory pVT/VF showing significant improvement in survival to hospital admission for the amiodarone group15. This was followed by the ALIVE trial comparing amiodarone and lidocaine which showed significantly higher rates of survival to hospital admission in the amiodarone group16. However, a more recent randomized trial comparing amiodarone (in a novel diluent less likely to cause hypotension), lidocaine, and placebo in a similar patient population showed less convincing results, with no detected difference in survival or the secondary outcome of favorable neurological outcome for either amiodarone or lidocaine compared with placebo17. The heterogeneity of available data contributed to current guidelines which recommend that either amiodarone or lidocaine may be used for shock-refractory pVT/VF18.

Current guidelines also recommend the administration of vasopressors (epinephrine 1mg every 3-5 minutes). In one randomized controlled trial exploring the long-standing guideline recommendations, epinephrine was associated with increased rates of restoration of spontaneous circulation, though no significant impact on the primary outcome of survival to hospital discharge was identified19. Physiologically, increased systemic vascular resistance combined with positive beta-adrenergic impact on cardiac output would be expected to complement resuscitative efforts. However, more recent studies have suggested that arrest physiology and unanticipated pharmacologic effects may complicate this simplistic interpretation – particularly when patient-centered outcomes are emphasized. Research exploring the timing and amount of epinephrine suggest that earlier administration and higher cumulative doses are associated with negative impacts on survival to hospital discharge and favorable neurological outcomes20-22.

Ultimately, treatment should focus on optimal execution of measures with clear benefits (namely chest compressions and early defibrillation of eligible rhythms). Other management considerations with which the emergency physician is familiar with including establishing peripheral access and definitive airway management can be delayed.

Rapid Diagnostic Measures for the Identification of Reversible Processes

Traditional diagnostic measures are generally unavailable during an ongoing cardiac arrest resuscitation. The emergency medicine physician must rely on the physical examination and point-of-care tests with the objective of identifying potentially reversible processes. Measurement of capillary blood glucose can exclude hypoglycemia as a contributor. Point-of-care chemistry and blood gas analyzers can identify important electrolyte derangements, as well as clarifying the primary impulse in acid-base disturbances.

End-tidal capnography was discussed previously for the guidance of ongoing resuscitation, but it may have diagnostic utility in patients with SCD. In one study the initial EtCO2 was noted to be significantly higher for primary pulmonary processes (with PEA/asystole as presenting rhythm) compared to primary cardiac processes (with pVT/VF as presenting rhythm)23.

The use of point-of-care ultrasonography, particularly in PEA arrest where non-cardiac etiologies dominate, may help identify the etiology of arrest and direct therapy. Bedside ultrasonography should be directed first at assessment of cardiac function – examining the pericardial sac and gross abnormalities in chamber size. A pericardial effusion may suggest cardiac tamponade, ventricular collapse can be seen with hypovolemia, and asymmetric right-ventricular dilation points to pulmonary embolus where thrombolysis should be considered8. If cardiac ultrasound is unrevealing, thoracic ultrasound can identify pneumothorax24-27.

In the absence of ultrasonographic abnormalities, attention turns to other rapidly reversible precipitants first. If opioid toxicity is a consideration, an attempt at reversal with naloxone has few adverse effects. If any detected rhythm is a polymorphic ventricular tachycardia characteristic of torsades de pointes – rapid infusion of magnesium sulfate should follow defibrillation. Other potentially reversible medications or toxins should be managed as appropriate.

Post-Resuscitation Steps

After successful restoration of circulation, the next management steps are critical to the patient’s long-term outcomes. A definitive airway should be established if not already secured (and if restoration of circulation was not associated with neurological recovery sufficient for independent airway protection). Circulatory support should continue with fluid resuscitation and vasopressors to maintain end-organ perfusion.

An immediate ECG should be performed to identify infarction, ischemia or precipitants of dysrhythmia. ST-segment elevation after return of spontaneous circulation (ROSC) warrants emergent angiography and possible intervention. However, given the prevalence of cardiac causes (of which coronary disease is most common) for patients with pVT/VF arrest, the presence of ST elevations is likely of insufficient sensitivity to identify all patients who would benefit from angiography. Several studies and meta-analyses have explored a more inclusive selection strategy for angiography (patients without obvious non-cardiac causes for arrest), all of which identified survival benefits with angiography and successful angioplasty when possible28-30.

Finally, the induction of hypothermia (or targeted temperature management) has significant benefits in survivors of cardiac arrest and can be instituted in the emergency department. Studies first targeted a core temperature of 32-24°C, with a randomized controlled trial demonstrating higher rates of favorable neurological outcome and reduced mortality31. More recent studies suggest that a more liberal temperature target does not diffuse the positive effects of induced hypothermia. A randomized trial of 939 patients with OHCA comparing a targeted temperature of 33°C vs 36°C suggested that a lower temperature target did not confer higher benefit to mortality or recovery of neurological function32. The more liberal temperature target may alleviate adverse effects associated with hypothermia which include cardiovascular effects (bradycardia), electrolyte derangements (during induction and rewarming), and possible increased risk of infections33. Targeted temperature management is achieved with external cooling measures or infusion of cooled fluids, rarely requiring more invasive measures34. Aggregate review of available data in a recent meta-analysis further supports the use of targeted temperature management after cardiac arrest as standard-of-care35.

References

  1. Bergum D, Nordseth T, Mjølstad OC, Skogvoll E, Haugen BO. Causes of in-hospital cardiac arrest – Incidences and rate of recognition. Resuscitation. 2015;87:63-68. doi:10.1016/j.resuscitation.2014.11.007.
  2. Wallmuller C, Meron G, Kurkciyan I, Schober A, Stratil P, Sterz F. Causes of in-hospital cardiac arrest and influence on outcome. Resuscitation. 2012;83(10):1206-1211. doi:10.1016/j.resuscitation.2012.05.001.
  3. Vaartjes I, Hendrix A, Hertogh EM, et al. Sudden death in persons younger than 40 years of age: incidence and causes. European Journal of Cardiovascular Prevention & Rehabilitation. 2009;16(5):592-596. doi:10.1097/HJR.0b013e32832d555b.
  4. Zheng ZJ, Croft JB, Giles WH, Mensah GA. Sudden cardiac death in the United States, 1989 to 1998. Circulation. 2001;104(18):2158-2163.
  5. Hoes AW, Grobbee DE, Lubsen J, Man in ‘t Veld AJ, van der Does E, Hofman A. Diuretics, beta-blockers, and the risk for sudden cardiac death in hypertensive patients. Ann Intern Med. 1995;123(7):481-487.
  6. Siscovick DS, Raghunathan TE, Psaty BM, et al. Diuretic therapy for hypertension and the risk of primary cardiac arrest. N Engl J Med. 1994;330(26):1852-1857. doi:10.1056/NEJM199406303302603.
  7. Gettes LS. Electrolyte abnormalities underlying lethal and ventricular arrhythmias. Circulation. 1992;85(1 Suppl):I70-I76.
  8. Kürkciyan I, Meron G, Sterz F, et al. Pulmonary embolism as a cause of cardiac arrest: presentation and outcome. Arch Intern Med. 2000;160(10):1529-1535.
  9. Callaway CW, Soar J, Aibiki M, et al. Part 4: Advanced Life Support: 2015 International Consensus on Cardiopulmonary Resuscitation and Emergency Cardiovascular Care Science With Treatment Recommendations. In: Vol 132. American Heart Association, Inc.; 2015:S84-S145. doi:10.1161/CIR.0000000000000273.
  10. Kudenchuk PJ, Redshaw JD, Stubbs BA, et al. Impact of changes in resuscitation practice on survival and neurological outcome after out-of-hospital cardiac arrest resulting from nonshockable arrhythmias. Circulation. 2012;125(14):1787-1794. doi:10.1161/CIRCULATIONAHA.111.064873.
  11. Touma O, Davies M. The prognostic value of end tidal carbon dioxide during cardiac arrest: a systematic review. Resuscitation. 2013;84(11):1470-1479. doi:10.1016/j.resuscitation.2013.07.011.
  12. Levine RL, Wayne MA, Miller CC. End-tidal carbon dioxide and outcome of out-of-hospital cardiac arrest. N Engl J Med. 1997;337(5):301-306. doi:10.1056/NEJM199707313370503.
  13. Garnett AR, Ornato JP, Gonzalez ER, Johnson EB. End-tidal carbon dioxide monitoring during cardiopulmonary resuscitation. JAMA. 1987;257(4):512-515.
  14. Falk JL, Rackow EC, Weil MH. End-tidal carbon dioxide concentration during cardiopulmonary resuscitation. N Engl J Med. 1988;318(10):607-611. doi:10.1056/NEJM198803103181005.
  15. Kudenchuk PJ, Cobb LA, Copass MK, et al. Amiodarone for resuscitation after out-of-hospital cardiac arrest due to ventricular fibrillation. N Engl J Med. 1999;341(12):871-878. doi:10.1056/NEJM199909163411203.
  16. Dorian P, Cass D, Schwartz B, Cooper R, Gelaznikas R, Barr A. Amiodarone as compared with lidocaine for shock-resistant ventricular fibrillation. N Engl J Med. 2002;346(12):884-890. doi:10.1056/NEJMoa013029.
  17. Kudenchuk PJ, Brown SP, Daya M, et al. Amiodarone, Lidocaine, or Placebo in Out-of-Hospital Cardiac Arrest. N Engl J Med. 2016;374(18):1711-1722. doi:10.1056/NEJMoa1514204.
  18. Neumar RW, Shuster M, Callaway CW, et al. Part 1: Executive Summary: 2015 American Heart Association Guidelines Update for Cardiopulmonary Resuscitation and Emergency Cardiovascular Care. In: Vol 132. American Heart Association, Inc.; 2015:S315-S367. doi:10.1161/CIR.0000000000000252.
  19. Jacobs IG, Finn JC, Jelinek GA, Oxer HF, Thompson PL. Effect of adrenaline on survival in out-of-hospital cardiac arrest: A randomised double-blind placebo-controlled trial. Resuscitation. 2011;82(9):1138-1143. doi:10.1016/j.resuscitation.2011.06.029.
  20. Hagihara A, Hasegawa M, Abe T, Nagata T, Wakata Y, Miyazaki S. Prehospital epinephrine use and survival among patients with out-of-hospital cardiac arrest. JAMA. 2012;307(11):1161-1168. doi:10.1001/jama.2012.294.
  21. Dumas F, Bougouin W, Geri G, et al. Is epinephrine during cardiac arrest associated with worse outcomes in resuscitated patients? J Am Coll Cardiol. 2014;64(22):2360-2367. doi:10.1016/j.jacc.2014.09.036.
  22. Andersen LW, Kurth T, Chase M, et al. Early administration of epinephrine (adrenaline) in patients with cardiac arrest with initial shockable rhythm in hospital: propensity score matched analysis. BMJ. 2016;353:i1577. doi:10.1136/bmj.i1577.
  23. Grmec S, Lah K, Tusek-Bunc K. Difference in end-tidal CO2 between asphyxia cardiac arrest and ventricular fibrillation/pulseless ventricular tachycardia cardiac arrest in the prehospital setting. Crit Care. 2003;7(6):R139-R144. doi:10.1186/cc2369.
  24. Rose JS, Bair AE, Mandavia D, Kinser DJ. The UHP ultrasound protocol: a novel ultrasound approach to the empiric evaluation of the undifferentiated hypotensive patient. American Journal of Emergency Medicine. 2001;19(4):299-302. doi:10.1053/ajem.2001.24481.
  25. Hernandez C, Shuler K, Hannan H, Sonyika C, Likourezos A, Marshall J. C.A.U.S.E.: Cardiac arrest ultra-sound exam—A better approach to managing patients in primary non-arrhythmogenic cardiac arrest. Resuscitation. 2008;76(2):198-206. doi:10.1016/j.resuscitation.2007.06.033.
  26. Chardoli M, Heidari F, Shuang-ming S, et al. Echocardiography integrated ACLS protocol versus con- ventional cardiopulmonary resuscitation in patients with pulseless electrical activity cardiac arrest. Chinese Journal of Traumatology. 2012;15(5):284-287. doi:10.3760/cma.j.issn.1008-1275.2012.05.005.
  27. Zengin S, Yavuz E, Al B, et al. Benefits of cardiac sonography performed by a non-expert sonographer in patients with non-traumatic cardiopulmonary arrest. Resuscitation. 2016;102:105-109. doi:10.1016/j.resuscitation.2016.02.025.
  28. Spaulding CM, Joly LM, Rosenberg A, et al. Immediate coronary angiography in survivors of out-of-hospital cardiac arrest. N Engl J Med. 1997;336(23):1629-1633. doi:10.1056/NEJM199706053362302.
  29. Dumas F, Cariou A, Manzo-Silberman S, et al. Immediate Percutaneous Coronary Intervention Is Associated With Better Survival After Out-of-Hospital Cardiac Arrest: Insights From the PROCAT (Parisian Region Out of Hospital Cardiac Arrest) Registry. Circulation: Cardiovascular Interventions. 2010;3(3):200-207. doi:10.1161/CIRCINTERVENTIONS.109.913665.
  30. Millin MG, Comer AC, Nable JV, et al. Patients without ST elevation after return of spontaneous circulation may benefit from emergent percutaneous intervention: A systematic review and meta-analysis. Resuscitation. 2016;108:54-60. doi:10.1016/j.resuscitation.2016.09.004.
  31. Hypothermia after Cardiac Arrest Study Group. Mild therapeutic hypothermia to improve the neurologic outcome after cardiac arrest. N Engl J Med. 2002;346(8):549-556. doi:10.1056/NEJMoa012689.
  32. Nielsen N, Wetterslev J, Cronberg T, et al. Targeted Temperature Management at 33°C versus 36°C after Cardiac Arrest. N Engl J Med. 2013;369(23):2197-2206. doi:10.1056/NEJMoa1310519.
  33. Polderman KH, Peerdeman SM, Girbes AR. Hypophosphatemia and hypomagnesemia induced by cooling in patients with severe head injury. J Neurosurg. 2001;94(5):697-705. doi:10.3171/jns.2001.94.5.0697.
  34. Polderman KH, Herold I. Therapeutic hypothermia and controlled normothermia in the intensive care unit: Practical considerations, side effects, and cooling methods*. Critical Care Medicine. 2009;37(3):1101-1120. doi:10.1097/CCM.0b013e3181962ad5.
  35. Schenone AL, Cohen A, Patarroyo G, et al. Therapeutic hypothermia after cardiac arrest: A systematic review/meta-analysis exploring the impact of expanded criteria and targeted temperature. Resuscitation. 2016;108:102-110. doi:10.1016/j.resuscitation.2016.07.238.

Atrial Fibrillation

A 64-year-old male with a history of hypertension and hyperlipidemia presents with palpitations. He reports intermittent symptoms over the past 3 days, associated with dyspnea on exertion, but no chest pain, dizziness or syncope. His vital signs are notable for tachycardia (HR 129bpm) without hypotension or hypoxia. An ECG shows atrial fibrillation with rapid ventricular response.

Evaluation and Management

While the patient’s tachyarrhythmia is not yet associated with hypotension or evidence of malperfusion, preparation is key and includes continuous telemetry and vital sign monitoring, establishment of intravenous access, and application of cardioversion pads 1.

The presence of atrial fibrillation is new and the rapid ventricular response (RVR) may be symptomatic of more serious and potentially reversible pathology. A thorough history and physical examination may elucidate a precipitant and should precede attempts at rate- or rhythm-control. RVR may be provoked by any of the processes that would otherwise induce a sinus tachycardia, including bleeding, infection, toxic/metabolic etiologies and endocrinopathies 2, 3

Candidacy for Cardioversion

In hemodynamically stable patients with new-onset atrial fibrillation, candidacy for cardioversion includes:4-6

  • Stable without ischemia, hypotension or acute CHF
  • Clear onset of <48 hours
  • Non-severe symptoms
  • Few prior episodes/treatments
  • Existing anti-coagulation with warfarin and therapeutic INR (at least 3 weeks)
  • Absence of high-risk features: rheumatic/valvular disease, severe left-ventricular dysfunction, prosthetic valves, or history of thromboembolism

Cardioversion may be pharmacologic (with procainamide, or amiodarone), or electrical (synchronized at 100-200J). Electrical cardioversion for acute atrial fibrillation is both more effective and results in shorter lengths-of-stay in the emergency department – though stable patients should participate in shared decision-making7. Another important consideration when cardioversion is pursued is the prevention of systemic embolization. While atrial fibrillation of duration less than 48-hours is rarely associated with systemic embolization, certain populations are at higher risk8. One retrospective study of 3143 patients with atrial fibrillation for less than 48-hours demonstrated an overall risk of 0.7% for thromboembolic events – though the rate was significantly higher in patients older than 60 years or with other comorbidities (heart failure, diabetes)9. The risk of embolic events should be weighed against the risk of bleeding.

CHA2DS2VASc10-12

C Congestive Heart Failure 1
H Hypertension 1
A2 Age >75 2
D Diabetes Mellitus 1
S2 Stroke, TIA, Thromboembolism 2
V Vascular disease 1
A Age >65 1
Sc Sex Category Female 1

 

  • 0: low risk (may not require anti-coagulation)
  • 1: low-moderate risk (consider anti-platelet or anti-coagulation)
  • ≥ 2: moderate-high risk (anti-coagulation recommended)

HAS-BLED13,14

H Uncontrolled hypertension 1
A Abnormal renal/liver function
Renal (renal replacement therapy, creatinine >2.3mg/dL) 1
Liver (cirrhosis, bilirubin >2x, AST/ALT >3x) 1
S Stroke 1
B Bleeding history/anemia 1
L Labile INR 1
E Elderly (>65) 1
D Drugs
Anti-platelet agent, NSAID 1
Alcohol (>8 drinks/week) 1

 

  • 0: low risk (0.6-1.13% annual risk of major bleeding)
  • 1-2: intermediate risk (1.02-3.2% annual risk of major bleeding)
  • ≥ 3: high risk (4.9-19.6% annual risk of major bleeding)

Pharmacologic Management

For patients who are not candidates for cardioversion, rate-control should be pursued. Options include AV nodal blocking agents such as calcium channel blockers and beta-blockers15. The most frequently studied agents of each category are metoprolol and diltiazem. Both classes show comparable efficacy and safety profiles with trends favoring diltiazem16, 17.

Algorithm for the management of atrial fibrillation with rapid ventricular response:

Algorithm for the management of atrial fibrillation with rapid ventricular response

References

  1. Atzema, C.L. and T.W. Barrett, Managing atrial fibrillation. Ann Emerg Med, 2015. 65(5): p. 532-9.
  2. January, C.T., et al., 2014 AHA/ACC/HRS Guideline for the Management of Patients With Atrial Fibrillation: Executive Summary. A Report of the American College of Cardiology/American Heart Association Task Force on Practice Guidelines and the Heart Rhythm Society, 2014. 64(21): p. 2246-2280.
  3. Scheuermeyer, F.X., et al., Emergency Department Patients With Atrial Fibrillation or Flutter and an Acute Underlying Medical Illness May Not Benefit From Attempts to Control Rate or Rhythm. Ann Emerg Med, 2015. 65(5): p. 511-522 e2.
  4. Stiell, I.G., et al., Association of the Ottawa Aggressive Protocol with rapid discharge of emergency department patients with recent-onset atrial fibrillation or flutter. CJEM, 2010. 12(3): p. 181-91.
  5. von Besser, K. and A.M. Mills, Is discharge to home after emergency department cardioversion safe for the treatment of recent-onset atrial fibrillation? Ann Emerg Med, 2011. 58(6): p. 517-20.
  6. Cohn, B.G., S.M. Keim, and D.M. Yealy, Is emergency department cardioversion of recent-onset atrial fibrillation safe and effective? J Emerg Med, 2013. 45(1): p. 117-27.
  7. Bellone, A., et al., Cardioversion of acute atrial fibrillation in the emergency department: a prospective randomised trial. Emerg Med J, 2012. 29(3): p. 188-91.
  8. Weigner, M.J., et al., Risk for clinical thromboembolism associated with conversion to sinus rhythm in patients with atrial fibrillation lasting less than 48 hours. Ann Intern Med, 1997. 126(8): p. 615-20.
  9. Airaksinen, K.E., et al., Thromboembolic complications after cardioversion of acute atrial fibrillation: the FinCV (Finnish CardioVersion) study. J Am Coll Cardiol, 2013. 62(13): p. 1187-92.
  10. Friberg, L., M. Rosenqvist, and G.Y. Lip, Evaluation of risk stratification schemes for ischaemic stroke and bleeding in 182 678 patients with atrial fibrillation: the Swedish Atrial Fibrillation cohort study. Eur Heart J, 2012. 33(12): p. 1500-10.
  11. Lip, G.Y., et al., Refining clinical risk stratification for predicting stroke and thromboembolism in atrial fibrillation using a novel risk factor-based approach: the euro heart survey on atrial fibrillation. Chest, 2010. 137(2): p. 263-72.
  12. Ntaios, G., et al., CHADS(2), CHA(2)S(2)DS(2)-VASc, and long-term stroke outcome in patients without atrial fibrillation. Neurology, 2013. 80(11): p. 1009-17.
  13. Lip, G.Y., et al., Comparative validation of a novel risk score for predicting bleeding risk in anticoagulated patients with atrial fibrillation: the HAS-BLED (Hypertension, Abnormal Renal/Liver Function, Stroke, Bleeding History or Predisposition, Labile INR, Elderly, Drugs/Alcohol Concomitantly) score. J Am Coll Cardiol, 2011. 57(2): p. 173-80.
  14. Pisters, R., et al., A novel user-friendly score (HAS-BLED) to assess 1-year risk of major bleeding in patients with atrial fibrillation: the Euro Heart Survey. Chest, 2010. 138(5): p. 1093-100.
  15. Goralnick, E. and L.J. Bontempo, Atrial Fibrillation. Emerg Med Clin North Am, 2015. 33(3): p. 597-612.
  16. Demircan, C., et al., Comparison of the effectiveness of intravenous diltiazem and metoprolol in the management of rapid ventricular rate in atrial fibrillation. Emerg Med J, 2005. 22(6): p. 411-4.
  17. Fromm, C., et al., Diltiazem vs. Metoprolol in the Management of Atrial Fibrillation or Flutter with Rapid Ventricular Rate in the Emergency Department. J Emerg Med, 2015. 49(2): p. 175-82.
  18. DiMarco, J.P., Atrial fibrillation and acute decompensated heart failure. Circ Heart Fail, 2009. 2(1): p. 72-3.

Febrile Seizure

Brief HPI:

An 8-month old female, fully-immunized, otherwise healthy is brought in by paramedics after 1 minute of witnessed generalized tonic-clonic shaking. The patient had otherwise been well, eating and behaving normally earlier that day. On EMS arrival, the patient was post-ictal but grew increasingly responsive en-route and upon presentation to the pediatric emergency department she was crying and appeared normal to her parents. Capillary glucose was 118g/dL. On examination the patient was noted to be febrile with a rectal temperature of 39.4°C. The remainder of the physical examination was normal.

ED Course:

The patient received anti-pyretics and a urinalysis was obtained which was not suggestive of urinary tract infection. During the 3-hour period of observation in the emergency department the patient remained at her normal baseline, had no further seizure activity, and tolerated oral intake with difficulty. The patient was suspected to have a simple febrile seizure and was discharged home.

Algorithm for the Diagnosis of Febrile Seizure

Algorithm for the Evaluation of Febrile Seizure

References

  1. Syndi Seinfeld DO, Pellock JM. Recent Research on Febrile Seizures: A Review. J Neurol Neurophysiol. 2013;4(165). doi:10.4172/2155-9562.1000165.
  2. Whelan H, Harmelink M, Chou E, et al. Complex febrile seizures-A systematic review. Dis Mon. 2017;63(1):5-23. doi:10.1016/j.disamonth.2016.12.001.
  3. Millichap JJ, Gordon Millichap J. Methods of investigation and management of infections causing febrile seizures. Pediatr Neurol. 2008;39(6):381-386. doi:10.1016/j.pediatrneurol.2008.07.017.
  4. Subcommittee on Febrile Seizures, American Academy of Pediatrics. Neurodiagnostic evaluation of the child with a simple febrile seizure. Pediatrics. 2011;127(2):389-394. doi:10.1542/peds.2010-3318.

Leukemoid Reaction

Brief HPI:

An approximately 80-year-old male with unknown medical history is brought to the emergency department from a skilled nursing facility after unwitnessed arrest – EMS providers established return of spontaneous circulation after chest compressions and epinephrine. On arrival, the patient was hypotensive (MAP 40mmHg) and hypoxic (SpO2 85%) with mask ventilation. The patient was intubated, resuscitated with intravenous fluids and started on vasopressors. Imaging demonstrated lung consolidation consistent with multifocal pneumonia versus aspiration. Laboratory studies were obtained:

  • CBC: WBC: 49.2 (N: 64%, Bands: 20%)
  • ABG: pH: 7.07, pCO2: 73mmHg
  • Lactate: 9.1mmol/L
leukemoid_ct_01
leukemoid_ct_02
leukemoid_ct_03
leukemoid_ct_04
leukemoid_ct_05
leukemoid_ct_06
leukemoid_ct_07
leukemoid_ct_08
leukemoid_ct_09
leukemoid_ct_10
leukemoid_ct_11
leukemoid_ct_12

CT Pulmonary Angiography

Peribronchial opacities and patchy consolidation in the lungs which may represent multifocal pneumonia and/or aspiration in the appropriate clinical setting.
Mildly dilated main pulmonary artery suggestive of pulmonary arterial hypertension.

ED Course:

The patient was admitted to the medical intensive care unit for cardiopulmonary arrest presumed secondary to hypoxia and septic shock from healthcare-associated pneumonia or aspiration. The markedly elevated white blood cell count was attributed to a combination of infection and tissue ischemia from transient global hypoperfusion.


Definition: 1

  • Markedly elevated leukocyte (particularly neutrophil) count without hematologic malignancy
  • Cutoff is variable, 25-50k

Review of Available Literature

Retrospective review of 135 patients with WBC >25k 2
48% infection
15% malignancy
9% hemorrhage
12% glucocorticoid or granulocyte colony stimulating therapy
Retrospective review of 173 patients with WBC >30k 3
48% infection (7% C. difficile)
28% tissue ischemia
7% obstetric process (vaginal or cesarean delivery)
5% malignancy
Observational study of 54 patients with WBC >25k 4
Consecutive patients presenting to the emergency department
Compared to age-matched controls with moderate leukocytosis (12-24k)
Patients with leukemoid reaction were more likely to have an infection, be hospitalized and die.

Differential Diagnosis of Leukemoid Reaction 1,5-8

Differential Diagnosis of Leukemoid Reaction

References

  1. Sakka V, Tsiodras S, Giamarellos-Bourboulis EJ, Giamarellou H. An update on the etiology and diagnostic evaluation of a leukemoid reaction. Eur J Intern Med. 2006;17(6):394-398. doi:10.1016/j.ejim.2006.04.004.
  2. Reding MT, Hibbs JR, Morrison VA, Swaim WR, Filice GA. Diagnosis and outcome of 100 consecutive patients with extreme granulocytic leukocytosis. Am J Med. 1998;104(1):12-16.
  3. Potasman I, Grupper M. Leukemoid reaction: spectrum and prognosis of 173 adult patients. Clin Infect Dis. 2013;57(11):e177-e181. doi:10.1093/cid/cit562.
  4. Lawrence YR, Raveh D, Rudensky B, Munter G. Extreme leukocytosis in the emergency department. QJM. 2007;100(4):217-223. doi:10.1093/qjmed/hcm006.
  5. Marinella MA, Burdette SD, Bedimo R, Markert RJ. Leukemoid reactions complicating colitis due to Clostridium difficile. South Med J. 2004;97(10):959-963. doi:10.1097/01.SMJ.0000054537.20978.D4.
  6. Okun DB, Tanaka KR. Profound leukemoid reaction in cytomegalovirus mononucleosis. JAMA. 1978;240(17):1888-1889.
  7. Halkes CJM, Dijstelbloem HM, Eelkman Rooda SJ, Kramer MHH. Extreme leucocytosis: not always leukaemia. Neth J Med. 2007;65(7):248-251.
  8. Granger JM, Kontoyiannis DP. Etiology and outcome of extreme leukocytosis in 758 nonhematologic cancer patients: a retrospective, single-institution study. Cancer. 2009;115(17):3919-3923. doi:10.1002/cncr.24480.

Thrombocytopenia

Brief HPI:

A middle-aged female with no known medical history is brought to the emergency department with altered mental status. Her family notes worsening confusion over the past 2-3 days associated with vomiting and yellow discoloration of skin and eyes.

Initial vital signs were normal, though with borderline hypotension (99/64mmHg). Examination demonstrated an alert, but lethargic patient with jaundice and scleral icterus, no skin lesions were appreciated. Laboratory studies were obtained:

CBC

  • WBC: 21.3 (N: 83%, Bands: 11%)
  • Hb: 5.5
  • Plt: 6k
  • Marked schistocytes

Coagulation Panel

  • INR: 1.26
  • PTT: Normal
  • Fibrinogen: Normal
  • FDP: Normal
  • D-dimer: >9,000 (normal 250)
  • Haptoglobin: Undetectable
  • LDH: 1493

CMP

  • Creatinine: 1.1
  • AST/ALT: Normal
  • TB: 4.3, DB: 0.8

Imaging:

CT Head: No acute intracranial process.

ttp_ct_01
ttp_ct_02
ttp_ct_03
ttp_ct_04
ttp_ct_05
ttp_ct_06
ttp_ct_07
ttp_ct_08
ttp_ct_09
ttp_ct_10
ttp_ct_11
ttp_ct_12
ttp_ct_13
ttp_ct_14
ttp_ct_15
ttp_ct_16
ttp_ct_17
ttp_ct_18
ttp_ct_19

CT Abdomen/Pelvis with Contrast

Moderate free intra-abdominal fluid, heterogeneous liver with periportal edema, dense right middle lobe consolidation.

ED Course:

The patient developed worsening respiratory failure with hypoxia and tachypnea requiring endotracheal intubation. Thrombotic thrombocytopenic purpura was suspected and while awaiting emergent plasma exchange transfusion, the patient arrested and resuscitation efforts were unsuccessful.

The patient’s ADAMTS13 activity level was <3%. Autopsy demonstrated consolidation of the right middle lobe with possible lymphoproliferative mass, and lung petechial hemorrhages from microvascular thrombi.

Differential Diagnosis of Thrombocytopenia 1-7

Differential Diagnosis of Thrombocytopenia

Algorithm for the Evaluation of Thrombocytopenia 8

Algorithm for the Evaluation of Thrombocytopenia

Definition 9

  • Mild: <150k
  • Moderate: 100-150k
  • Severe: <50k
    • 10-30k: bleeding with minimal trauma
    • <10k: increased risk spontaneous bleeding

History 9,10

  • Prior platelet count
  • Family history bleeding disorders
  • Medications
    • Heparin
    • Quinine, quinidine
    • Rifampin
    • Trimethoprim-sulfamethoxazole
    • Vancomycin
  • Alcohol use
  • Travel-related infections

Physical Examination 9,10

  • Splenomegaly (liver disease)
  • Lymphadenopathy (infection, malignancy)

Workup 10,11

Schistocytes

Red blood cell fragments (schistocytes) 11

  • hCG
  • Repeat CBC
    • Detect spurious measure
    • Neutrophil-predominant leukocytosis: bacterial infection
    • Immature leukocytes (blasts): leukemia, myelodysplasia
  • Peripheral smear
    • Schistocytes: microangiopathic process (DIC, TTP, HUS)
    • Atypical lymphocytes: viral infection
    • Intracellular parasites: malaria
    • Hypersegmented neutrophils: nutritional deficiency
  • Infectious features: HIV, HCV, EBV, H.pylori, blood cultures
  • Autoimmune features: ANA, APL-Ab
  • Suspected occult liver disease: LFT, PT/PTT/INR
  • Suspected thrombotic microangiopathy: PT/PTT/INR, haptoglobin, LDH, fibrinogen, FDP, d-dimer

Specific Conditions 2-6,9,12-20

Disease Cause Presentation Laboratory Findings Treatment
DIC Sepsis
Trauma
Burn
Malignancy
Bleeding
Multi-organ failure
Shock
INR
Fibrinogen
FDP
D-dimer
Directed at underlying cause
Transfusion thresholds for hemorrhage:
FFP for INR >1.5
Platelets if <50k
Cryoprecipitate of fibrinogen <100mg/dL
TTP Insufficient ADAMTS-13 activity Non-specific constitutional symptoms (ex. weakness)
Neuro: headache, AMS, focal neuro deficit
GI: abdominal pain, nausea/vomiting
LDH
Reticulocyte
Unconjugated bilirubin
Haptoglobin
Plasma exchange
HUS Shiga-toxin-producing bacteria, E. coli O157:H7 Bloody diarrhea, anuria, oliguria, and hypertension Aggressive supportive care
HELLP Spectrum of eclampsia Hypertension
Visual symptoms
Headache
RUQ abdominal pain
AST/ALT
Uric acid
Unconjugated bilirubin
LDH
Reticulocyte
Haptoglobin
Delivery, MgSO4
ITP Primary ITP

Secondary ITP
– Drug
– Autoimmune
– Infection
– Malignancy

Usually asymptomatic, may have petechiae or easy bruising Isolated thrombocytopenia Steroids
HIT Exposure to heparin or LMWH Thrombocytopenia or a 50 percent reduction in platelet count between 5-10d exposure
New thrombosis or skin necrosis
4 T’s score
Platelet factor 4 antibodies Withdraw heparin

References

  1. Greinacher A, Selleng S. How I evaluate and treat thrombocytopenia in the intensive care unit patient. Blood. 2016;128(26):3032-3042. doi:10.1182/blood-2016-09-693655.
  2. Joly BS, Coppo P, Veyradier A. Thrombotic thrombocytopenic purpura. Blood. 2017;129(21):2836-2846. doi:10.1182/blood-2016-10-709857.
  3. Leslie SD, Toy PT. Laboratory hemostatic abnormalities in massively transfused patients given red blood cells and crystalloid. Am J Clin Pathol. 1991;96(6):770-773.
  4. Neunert C, Lim W, Crowther M, et al. The American Society of Hematology 2011 evidence-based practice guideline for immune thrombocytopenia. Blood. 2011;117(16):4190-4207. doi:10.1182/blood-2010-08-302984.
  5. Kappler S, Ronan-Bentle S, Graham A. Thrombotic microangiopathies (TTP, HUS, HELLP). Emerg Med Clin North Am. 2014;32(3):649-671. doi:10.1016/j.emc.2014.04.008.
  6. Greinacher A. Heparin-Induced Thrombocytopenia. Solomon CG, ed. N Engl J Med. 2015;373(3):252-261. doi:10.1056/NEJMcp1411910.
  7. Reardon JE Jr., Marques MB. Evaluation of Thrombocytopenia. Lab Med. 2006;37(4):248-250. doi:10.1309/R7P79KERAJHPRHLT.
  8. Stasi R. How to approach thrombocytopenia. Hematology Am Soc Hematol Educ Program. 2012;2012:191-197. doi:10.1182/asheducation-2012.1.191.
  9. Gauer RL, Braun MM. Thrombocytopenia. Am Fam Physician. 2012;85(6):612-622.
  10. Abrams CS. 172 – Thrombocytopenia. Twenty Fifth Edition. Elsevier Inc.; 2016:1159–1167.e2. doi:10.1016/B978-1-4557-5017-7.00172-0.
  11. Wilson CS, Vergara-Lluri ME, Brynes RK. Chapter 11 – Evaluation of Anemia, Leukopenia, and Thrombocytopenia. Second Edition. Elsevier Inc.; 2017:195-234.e195. doi:10.1016/B978-0-323-29613-7.00011-9.
  12. Hui P, Cook DJ, Lim W, Fraser GA, Arnold DM. The frequency and clinical significance of thrombocytopenia complicating critical illness: a systematic review. Chest. 2011;139(2):271-278. doi:10.1378/chest.10-2243.
  13. Jokiranta TS. HUS and atypical HUS. Blood. 2017;129(21):2847-2856. doi:10.1182/blood-2016-11-709865.
  14. Neunert CE. Management of newly diagnosed immune thrombocytopenia: can we change outcomes? Hematology Am Soc Hematol Educ Program. 2017;2017(1):400-405. doi:10.1182/asheducation-2017.1.400.
  15. Lambert MP, Gernsheimer TB. Clinical updates in adult immune thrombocytopenia. Blood. 2017;129(21):2829-2835. doi:10.1182/blood-2017-03-754119.
  16. Arepally GM. Heparin-induced thrombocytopenia. Blood. 2017;129(21):2864-2872. doi:10.1182/blood-2016-11-709873.
  17. Aster RH, Bougie DW. Drug-induced immune thrombocytopenia. N Engl J Med. 2007;357(6):580-587. doi:10.1056/NEJMra066469.
  18. Boral BM, Williams DJ, Boral LI. Disseminated Intravascular Coagulation. Am J Clin Pathol. 2016;146(6):670-680. doi:10.1093/ajcp/aqw195.
  19. Scully M, Hunt BJ, Benjamin S, et al. Guidelines on the diagnosis and management of thrombotic thrombocytopenic purpura and other thrombotic microangiopathies. Br J Haematol. 2012;158(3):323-335. doi:10.1111/j.1365-2141.2012.09167.x.
  20. Levine RL, Hursting MJ, Drexler A, Lewis BE, Francis JL. Heparin-induced thrombocytopenia in the emergency department. Ann Emerg Med. 2004;44(5):511-515. doi:10.1016/j.annemergmed.2004.06.004.

Hepatobiliary Ultrasound

Brief H&P:

A 43-year-old female with a history of hypertension, diabetes and obesity presents with right-upper quadrant abdominal pain for the past 1 week. The pain is characterized as burning, non-radiating, intermittent (with episodes lasting 10-30 minutes), resolving spontaneously and without apparent provoking features. She notes nausea but no vomiting, no changes in bowel or urinary habits. She similarly denies fevers, chest pain or shortness of breath. Vital signs were normal, and physical examination was notable only for right upper quadrant tenderness to palpation without rigidity or guarding.

An ECG demonstrates normal sinus rhythm, laboratory tests including liver function tests and lipase were normal and a bedside ultrasound of the right upper quadrant was performed demonstrating gallstones and a positive sonographic Murphy sign. The patient was diagnosed with acute cholecystitis, antibiotics were initiated, the patient was maintained NPO while general surgery was consulted.

Evaluation of Right-Upper Quadrant Abdominal Pain

The initial evaluation of a patient presenting with right-upper quadrant (or adjacent) abdominal pain typically includes laboratory tests such as a complete blood count, chemistry panel, liver function tests and serum lipase. In patients at risk for atypical presentations for an acute coronary syndrome or with other concerning symptoms, electrocardiography and cardiac enzymes may be indicated.

The differential diagnosis is broad. A systematic approach proceeds anatomically from superficial to deeper structures centered around the site of maximal pain.

Skin

Skin

Herpes zoster, erysipelas, or cellulitis

Connective Tissue

Connective Tissue

Intercostal muscle strain, myositis, fasciitis

Bone

Bone

Rib contusion or fracture

Hepatobiliary

Hepatobiliary

Hepatitis (infectious, toxin-mediated), perihepatitis (Fitz-Hugh-Curtis), hepatic abscess, symptomatic cholelithiasis, acute cholecystitis, ascending cholangitis, pancreatitis

Gastric

Gastric

Peptic ulcer disease, gastroesophageal reflux, gastritis, gastroparesis

Small Bowel

Small Bowel

Duodenal ulcer, small bowel obstruction

Large Bowel

Large Bowel

Retrocecal appendicitis, inflammatory bowel disease

Genitourinary

Genitourinary

Pyelonephritis, ureterolithiasis

Referred

Referred

Acute coronary syndrome, lower-lobe pneumonia, pulmonary embolus

Ultrasound in the Evaluation of Right Upper Quadrant Abdominal Pain

The diagnosis is unlikely to be made based on laboratory tests alone 1. However, the addition of bedside ultrasound, particularly for the evaluation of gallbladder pathology, is both rapid and reliable 2-8. The algorithm below provides a pathway for the incorporation of bedside ultrasound of the right upper quadrant in the evaluation of suspected gallbladder disease.

Algorithm for the Use of Ultrasound in the Evaluation of Right Upper Quadrant Abdominal Pain

A normal-appearing gallbladder absent gallstones should prompt a traversal of the anatomic approach to the differential diagnosis detailed above. If gallstones are identified, the association with a positive sonographic Murphy sign is highly predictive of acute cholecystitis 2,5,6,9. Acute cholecystitis may be associated with inflammatory gallbladder changes such as wall-thickening (>3mm) or pericholecystic fluid 3,5,6,10-13. However, in the absence of cholelithiasis or a positive sonographic Murphy sign, these features are non-specific and may be the result of generalized edematous states such as congestive heart failure, renal failure, or hepatic failure and critically-ill patients may develop acalculous cholecystitis 7,11,14. Finally, common bile duct dilation may be due to intra-luminal obstruction as in choledocholithiasis, luminal abnormalities such as strictures, or extra-luminal compression from masses or malignancy.  Dilation is generally described as a diameter >6mm – allowing an additional 1mm for every decade over 60 years-old as well as more vague accommodations for patients with prior cholecystectomy 3,5,7,15.

Gallery

The POCUS Atlas
The ultrasound images and videos used in this post come from The POCUS Atlas, a collaborative collection focusing on rare, exotic and perfectly captured ultrasound images.
The POCUS Atlas

Gallstones

Many gallstones

Gallbladder wall thickening

Pericholecystic fluid

Choledocholithiasis

Common bile duct dilation

All illustrations are available for free, licensed (along with all content on this site) under Creative Commons Attribution-ShareAlike 4.0 International Public License.

Downloads Page License

References

  1. Trowbridge RL, Rutkowski NK, Shojania KG. Does this patient have acute cholecystitis? JAMA. 2003;289(1):80-86.
  2. Scruggs W, Fox JC, Potts B, et al. Accuracy of ED Bedside Ultrasound for Identification of gallstones: retrospective analysis of 575 studies. West J Emerg Med. 2008;9(1):1-5.
  3. Ross M, Brown M, McLaughlin K, et al. Emergency physician-performed ultrasound to diagnose cholelithiasis: a systematic review. Acad Emerg Med. 2011;18(3):227-235. doi:10.1111/j.1553-2712.2011.01012.x.
  4. Jang T, Chauhan V, Cundiff C, Kaji AH. Assessment of emergency physician-performed ultrasound in evaluating nonspecific abdominal pain. Am J Emerg Med. 2014;32(5):457-460. doi:10.1016/j.ajem.2014.01.004.
  5. Kendall JL, Shimp RJ. Performance and interpretation of focused right upper quadrant ultrasound by emergency physicians. J Emerg Med. 2001;21(1):7-13.
  6. Summers SM, Scruggs W, Menchine MD, et al. A prospective evaluation of emergency department bedside ultrasonography for the detection of acute cholecystitis. Ann Emerg Med. 2010;56(2):114-122. doi:10.1016/j.annemergmed.2010.01.014.
  7. Rubens DJ. Ultrasound Imaging of the Biliary Tract. Ultrasound Clinics. 2007;2(3):391-413. doi:10.1016/j.cult.2007.08.007.
  8. Rosen CL, Brown DF, Chang Y, et al. Ultrasonography by emergency physicians in patients with suspected cholecystitis. American Journal of Emergency Medicine. 2001;19(1):32-36. doi:10.1053/ajem.2001.20028.
  9. Shea JA. Revised Estimates of Diagnostic Test Sensitivity and Specificity in Suspected Biliary Tract Disease. Arch Intern Med. 1994;154(22):2573-2581. doi:10.1001/archinte.1994.00420220069008.
  10. Miller AH, Pepe PE, Brockman CR, Delaney KA. ED ultrasound in hepatobiliary disease. J Emerg Med. 2006;30(1):69-74. doi:10.1016/j.jemermed.2005.03.017.
  11. Shah K, Wolfe RE. Hepatobiliary ultrasound. Emergency Medicine Clinics of NA. 2004;22(3):661–73–viii. doi:10.1016/j.emc.2004.04.015.
  12. Matcuk GR, Grant EG, Ralls PW. Ultrasound measurements of the bile ducts and gallbladder: normal ranges and effects of age, sex, cholecystectomy, and pathologic states. Ultrasound Q. 2014;30(1):41-48. doi:10.1097/RUQ.0b013e3182a80c98.
  13. Engel JM, Deitch EA, Sikkema W. Gallbladder wall thickness: sonographic accuracy and relation to disease. American Journal of Roentgenology. 1980;134(5):907-909. doi:10.2214/ajr.134.5.907.
  14. Gerstenmaier JF, Hoang KN, Gibson RN. Contrast-enhanced ultrasound in gallbladder disease: a pictorial review. Abdom Radiol (NY). 2016;41(8):1640-1652. doi:10.1007/s00261-016-0729-4.
  15. Becker BA, Chin E, Mervis E, Anderson CL, Oshita MH, Fox JC. Emergency biliary sonography: utility of common bile duct measurement in the diagnosis of cholecystitis and choledocholithiasis. J Emerg Med. 2014;46(1):54-60. doi:10.1016/j.jemermed.2013.03.024.

Wide-complex Tachycardia

Several algorithms exist for the electrocardigraphic evaluation of regular, wide-complex tachycardias with the objective of distinguishing ventricular tachycardia (VT) from a supraventricular tachycardia (SVT) with aberrant conduction. The algorithm detailed below, developed by Dr. James Niemann, presents an ED-centric approach favoring the diagnosis of the more life-threatening dysrhythmia. This approach recognizes that SVT with aberrancy is rare, particularly in patients with a history of cardiac disease where the likelihood of ventricular tachycardia exceeds 90%. The algorithm requires the use of only the most simple and easily-recalled criteria, and any point of failure along the algorithm lends to the universally-appropriate management as ventricular tachycardia.

Algorithm for the Evaluation of Regular, Wide-Complex Tachycardia

Algorithm for the Evaluation of Wide-Complex Tachycardia

  1. aVR: Is the initial deflection in aVR positive? If yes, then VT.
  2. Concordance: Is there concordance (monophasic with same polarity) in all of the precordial leads? If yes, then VT.
  3. AV Dissociation: Is there evidence of AV dissociation (fusion or capture beats)? If yes, then VT.
  4. Bundle-branch morphology: Is the QRS morphology in V1 and V6 consistent with either LBBB or RBBB? If no, then VT.

References

  1. Neimann J. Wide QRS Complex Tachycardias. Lecture. Harbor-UCLA Department of Emergency Medicine. 2014:1-19.
  2. Vereckei A, Duray G, Szénási G, Altemose GT, Miller JM. New algorithm using only lead aVR for differential diagnosis of wide QRS complex tachycardia. Heart Rhythm. 2008;5(1):89-98. doi:10.1016/j.hrthm.2007.09.020.
  3. Szelényi Z, Duray G, Katona G, et al. Comparison of the “real-life” diagnostic value of two recently published electrocardiogram methods for the differential diagnosis of wide QRS complex tachycardias. Acad Emerg Med. 2013;20(11):1121-1130. doi:10.1111/acem.12247.
  4. Brugada P, Brugada J, Mont L, Smeets J, Andries EW. A new approach to the differential diagnosis of a regular tachycardia with a wide QRS complex. Circulation. 1991;83(5):1649-1659.
  5. Lau EW, Pathamanathan RK, Ng GA, Cooper J, Skehan JD, Griffith MJ. The Bayesian approach improves the electrocardiographic diagnosis of broad complex tachycardia. Pacing Clin Electrophysiol. 2000;23(10 Pt 1):1519-1526.
  6. B Garner J, M Miller J. Wide Complex Tachycardia – Ventricular Tachycardia or Not Ventricular Tachycardia, That Remains the Question. Arrhythm Electrophysiol Rev. 2013;2(1):23-29. doi:10.15420/aer.2013.2.1.23.
  7. Vereckei A. Current algorithms for the diagnosis of wide QRS complex tachycardias. Curr Cardiol Rev. 2014;10(3):262-276.
  8. Garmel GM. Wide Complex Tachycardias: Understanding this Complex Condition: Part 1 – Epidemiology and Electrophysiology. West J Emerg Med. 2008;9(1):28-39.
  9. Garmel GM. Wide Complex Tachycardias: Understanding this Complex Condition Part 2 – Management, Miscellaneous Causes, and Pitfalls. West J Emerg Med. 2008;9(2):97-103.
  10. Griffith MJ, Garratt CJ, Mounsey P, Camm AJ. Ventricular tachycardia as default diagnosis in broad complex tachycardia. The Lancet. 1994;343(8894):386-388.

Sinus Tachycardia

Brief History and Physical:

A young female with a history of schizophrenia presents to the emergency department reporting hallucinations. She had been diagnosed with schizophrenia one year previously and was briefly admitted to a psychiatric hospital. She discontinued her anti-psychotic (risperidone) two months ago, and over the past week she reports increasingly prominent auditory and visual hallucinations.

She denies recent illness, vomiting/diarrhea, changes in urinary habits, new medications, alcohol or illicit substance use. She also denies chest pain, palpitations or shortness of breath.

Vital signs are notable for a heart rate of 148bpm and are otherwise normal (including core temperature). Detailed physical examination is normal except for a rapid, regular heart rate. Mental status examination demonstrated normal level of alertness and orientation, linear and cogent responses and occasional response to internal stimuli during which she appeared anxious.

Initial evaluation and management included a 12-lead ECG which showed sinus tachycardia. Multiple boluses of normal saline were initiated while awaiting laboratory workup.

ECG: Sinus Tachycardia

Presentation ECG demonstrates sinus tachycardia.

Update:

Laboratory studies were reviewed and unremarkable. Normal hemoglobin, normal chemistry panel, negative hCG, and negative toxicology screen. The patient remained persistently tachycardic with a heart rate ranging from 140-160bpm (again sinus tachycardia on 12-lead ECG). An atypical antipsychotic and anxiolytic were administered and additional studies were obtained. Serum TSH, troponin and D-dimer were normal and bedside ultrasound did not identify a pericardial effusion. The patient remained asymptomatic, reporting subjective improvement in anxiety and hallucinations. Psychiatry was consulted and the patient was placed in observation for monitoring of sinus tachycardia. Observation course was uneventful as the patient remained asymptomatic. Transthoracic echocardiography was normal. Psychiatry consultation recommended resumption of home anti-psychotic and outpatient follow-up. Tachycardia had improved but not resolved at the time of discharge (heart rate 109bpm) and the patient was instructed to follow-up with her primary care provider.


Algorithm for the Evaluation of Sinus Tachycardia

Algorithm for the Evaluation of Sinus Tachycardia

Any vital sign derangement is concerning and tachycardia may be associated with unanticipated death after discharge home1. The presence of tachycardia suggests one of several categories of hemodynamic, autonomic, or endocrine/metabolic derangement.

Demand for increased cardiac output

A perceived demand for increased cardiac output will prompt chronotropic (and inotropic) amplification before hypotension develops. Causative etiologies include: volume depletion (from hemorrhage, gastrointestinal or renal losses), distributive processes (such as infection), obstruction (pulmonary embolus, or pericardial effusion with impending tamponade), or tissue hypoxia (anemia or lung disease).

Autonomic nervous system

Autonomic nervous system disturbances induced by stimulant, sympathomimetic or anti-cholinergic use, or withdrawal of certain agents such as ethanol or beta-blockers may be at fault.

Endocrine and other causes

Hyperthyroidism and pheochromocytoma should be considered, and as diagnoses of exclusion: anxiety, pain, or inappropriate sinus tachycardia2.

Evaluation:
Core temperature
CBC
Troponin
D-dimer
Bedside cardiac ultrasound
Urine toxicology screen
Ethanol level
TSH/T4

Algorithm for the Evaluation of Narrow-Complex Tachycardia3,4,5,6

Algorithm for the Evaluation of Narrow-Complex Tachycardia

References:

  1. Sklar DP, Crandall CS, Loeliger E, Edmunds K, Paul I, Helitzer DL. Unanticipated Death After Discharge Home From the Emergency Department. Ann Emerg Med. 2007;49(6):735-745. doi:10.1016/j.annemergmed.2006.11.018.
  2. Olshansky B, Sullivan RM. Inappropriate sinus tachycardia. J Am Coll Cardiol. 2013;61(8):793-801. doi:10.1016/j.jacc.2012.07.074.
  3. Yusuf S, Camm AJ. Deciphering the sinus tachycardias. Clin Cardiol. 2005;28(6):267-276.
  4. Katritsis DG, Josephson ME. Differential diagnosis of regular, narrow-QRS tachycardias. Heart Rhythm. 2015;12(7):1667-1676. doi:10.1016/j.hrthm.2015.03.046.
  5. Bibas L, Levi M, Essebag V. Diagnosis and management of supraventricular tachycardias. CMAJ. 2016;188(17-18):E466-E473. doi:10.1503/cmaj.160079.
  6. Link MS. Clinical practice. Evaluation and initial treatment of supraventricular tachycardia. N Engl J Med. 2012;367(15):1438-1448. doi:10.1056/NEJMcp1111259.

Tetanus Prophylaxis

An Algorithm for Tetanus Prophylaxis in Adults1

Algorithm for Tetanus Prophylaxis in Adults

References:

  1. Diphtheria, tetanus, and pertussis: recommendations for vaccine use and other preventive measures. Recommendations of the Immunization Practices Advisory committee (ACIP). MMWR Recomm Rep. 1991;40(RR-10):1-28.

ECG Guide: Pediatrics

ECG Standard

  • Full standard: no adjustment
  • Half-standard: commensurate reduction in amplitude (usually 50%)
  • Mixed: reduction in amplitude of precordial leads

Atrial Abnormalities

Right Atrial Abnormality (P pulmonale)
Peaked P-wave in II (>3mm from 0-6mo or >2.5mm >6mo)
Causes: right atrial volume overload, ASD, Ebstein, Fontan
Left Atrial Abnormality (P mitrale)
Wide, notched P-wave in II or biphasic in V1
Causes: MS, MR

Axis

  • Anatomical dominance of right ventricle until approximately 6mo
  • RAD normal
  • eRAD suggests AV canal defect

T-waves

  • 1st week of life: Upright
  • Adolescent: Inverted
  • Adult: Upright

Ventricular Hypertrophy

Right Ventricular Hypertrophy
R-wave height >98% for age in lead V1
S-wave depth >98% for age in lead V6
T-wave abnormality (ex. upright in childhood)
Causes: pHTN, PS, ToF
Left Ventricular Hypertrophy
R-wave height >98% for age in lead V6
S-wave depth >98% for age in lead V1
Adult-pattern R-wave progression in newborn (no large R-waves and small S-waves in right precordial leads)
Left-axis deviation
Causes: AS, coarctation, VSD, PDA

Examples


Normal Neonatal ECG

  • 2mo old
  • RAD
  • Inverted T-waves (normal)
  • Tall R-waves in V1-V3


Extreme Axis Deviation

  • Neonate with Down syndrome
  • Isoelectric in I, Negative in aVF negative in II  mean QRS vector -87°
  • Extreme RAD suggestive of AV canal defect


LVH:

  • Unrepaired Coarctation
  • Deep S-wave in V1 (>98%)
  • Tall R-wave in V6 (>98%)


RVH:

  • 10 year-old boy with pulmonary Hypertension
  • RAD after expected age for normal RAD
  • Tall R-waves in V1 (>98%)
  • Deep S-wave in V6 (>98%)


STEMI

  • ALCAPA (anomalous origin of the left coronary artery from the pulmonary artery): coronary artery arises anomalously from the pulmonary artery; as pulmonary arterial pressure falls during the first 6 months of infancy, prograde flow through the left coronary artery ceases and may even reverse.
  • HLHS (hypoplastic left heart syndrome): coronary arteries are perfused from a hypoplastic, narrow aorta that is susceptible to flow disruption
  • Orthotopic heart transplant with allograft vasculopathy
  • Kawasaki: coronary artery aneurysm with subsequent thrombosis


Benign early repolarization

  • 14 year-old male
  • Concave ST-segment elevation


Left Atrial Abnormality:

  • 9mo female with mitral insufficiency
  • Broad biphasic P-wave in V1
  • Tall, notched P-wave in II


Prolonged QT interval

  • 18-year-old female
  • Familial long QT syndrome and a history of cardiac arrest


WPW:

  • Delta wave, shortened PR interval

References

  1. O’Connor M, McDaniel N, Brady WJ. The pediatric electrocardiogram. Part I: Age-related interpretation. Am J Emerg Med. 2008;26(2):221-228. doi:10.1016/j.ajem.2007.08.003.
  2. Goodacre S, McLeod K. ABC of clinical electrocardiography: Paediatric electrocardiography. BMJ. 2002;324(7350):1382-1385.
  3. O’Connor M, McDaniel N, Brady WJ. The pediatric electrocardiogram Part II: Dysrhythmias. Am J Emerg Med. 2008;26(3):348-358. doi:10.1016/j.ajem.2007.07.034.
  4. O’Connor M, McDaniel N, Brady WJ. The pediatric electrocardiogram Part III: Congenital heart disease and other cardiac syndromes. Am J Emerg Med. 2008;26(4):497-503. doi:10.1016/j.ajem.2007.08.004.
  5. Schwartz P. Guidelines for the interpretation of the neonatal electrocardiogram. Eur Heart J. 2002;23(17):1329-1344. doi:10.1053/euhj.2002.3274.

Ultrasound in Dyspnea

Brief H&P:

A 68 year-old male with a history of hypertension, diabetes, hyperlipidemia, chronic obstructive pulmonary disease and congestive heart failure (CHF) with depressed ejection fraction presents via ambulance with a chief complaint of shortness of breath. EMS reports that the patient was tachypneic and saturating 80% on ambient air on their arrival. En route, he received nebulized albuterol, nitroglycerin and was started on non-invasive positive pressure ventilation (NI-PPV).

On arrival, he remains uncomfortable-appearing with a respiratory rate of 35 breaths/min and accessory muscle use. His heart rate is 136bpm, blood pressure is 118/85mmHg, and he is saturating 95% on an FiO2 of 100%. Attempts to obtain a history are limited due to difficulty comprehending his responses with the PPV mask on, and prompt desaturation with it off. Lung auscultation is similarly challenging due to ambient and transmitted sounds, although basilar crackles and diffuse expiratory wheezing are appreciated. Cardiovascular examination reveals a rapid and irregularly irregular rhythm. Assessment of jugular venous distension is limited due to the patient’s body habitus and the presence of mask straps around the patient’s neck. Lower extremities demonstrate 2+ pitting edema, symmetric bilaterally. Intravenous access is established and laboratory tests are sent. The ECG technician and portable chest x-ray are called.

The case presentation above demonstrates a common emergency department scenario: a critically-ill patient with undifferentiated dyspnea. Specifically, the scenario reveals a situation where the physical examination is either obfuscated by technical challenges or otherwise indeterminate. The patient is at risk for deterioration and targeted intervention is mandatory. If a COPD exacerbation is assumed, additional nebulized breathing treatments are indicated – a potentially costly jolt of beta agonists if the patient’s atrial fibrillation and rapid ventricular response are the consequence of decompensated systolic heart failure. Take the route of decompensated CHF and prompt afterload reduction with diuresis would be next – if incorrect, not only would the primary cause go untreated, but his tenuously-maintained blood pressure may suffer.

Algorithm for the Use of Ultrasound in the Evaluation of Dyspnea

Algorithm for the Use of Ultrasound in the Evaluation of Dyspnea

1. Lung Ultrasound

An approach incorporating point-of-care ultrasonography may be useful. First, a thoracic ultrasound is performed where certain causative etiologies might be identified immediately – for example absent lung sliding suggesting pneumothorax, or signs of generalized or subpleural consolidation.

The POCUS Atlas
The ultrasound images and videos used in this post come from The POCUS Atlas, a collaborative collection focusing on rare, exotic and perfectly captured ultrasound images.
The POCUS Atlas

Pneumothorax

Hepatization

Shred Sign

Pleural Effusion

2. Cardiac Ultrasound

Other findings on lung ultrasound may point to causes that are not primarily pulmonary. For example, if diffuse B-lines are encountered a focused cardiac ultrasound can be performed to grossly evaluate ejection fraction and estimate right atrial pressure.

B-Lines

Depressed EF

Dilated IVC

3. Venous Ultrasound

Finally, if the lung ultrasound is largely unremarkable (A-lines), a sequence of ultrasonographic findings including right ventricular dilation and the presence of a deep venous thrombosis would point to pulmonary embolism as the diagnosis.

DVT

RV Dilation

All illustrations are available for free, licensed (along with all content on this site) under Creative Commons Attribution-ShareAlike 4.0 International Public License.

Downloads Page License

References

  1. Lichtenstein DA, Mezière GA, Lagoueyte J-F, Biderman P, Goldstein I, Gepner A. A-lines and B-lines: lung ultrasound as a bedside tool for predicting pulmonary artery occlusion pressure in the critically ill. Chest. 2009;136(4):1014-1020. doi:10.1378/chest.09-0001.
  2. Copetti R, Soldati G, Copetti P. Chest sonography: a useful tool to differentiate acute cardiogenic pulmonary edema from acute respiratory distress syndrome. Cardiovasc Ultrasound. 2008;6(1):16. doi:10.1186/1476-7120-6-16.
  3. Gallard E, Redonnet J-P, Bourcier J-E, et al. Diagnostic performance of cardiopulmonary ultrasound performed by the emergency physician in the management of acute dyspnea. Am J Emerg Med. 2015;33(3):352-358. doi:10.1016/j.ajem.2014.12.003.
  4. Lichtenstein DA. Lung ultrasound in the critically ill. Ann Intensive Care. 2014;4(1):1. doi:10.1186/2110-5820-4-1.
  5. Zanobetti M, Scorpiniti M, Gigli C, et al. Point-of-Care Ultrasonography for Evaluation of Acute Dyspnea in the ED. Chest. 2017;151(6):1295-1301. doi:10.1016/j.chest.2017.02.003.
  6. Lichtenstein DA, Mezière GA. Relevance of lung ultrasound in the diagnosis of acute respiratory failure: the BLUE protocol. Chest. 2008;134(1):117-125. doi:10.1378/chest.07-2800.
  7. Images from The POCUS Atlas
  8. Special thanks to Dr. Timothy Jang, Director Emergency Ultrasound Program, Director Emergency Ultrasound Fellowship, Associate Professor of Clinical Emergency Medicine, Department of Emergency Medicine at Harbor-UCLA

ECG Guide: Part II

STEMI

STEMI

  • ST-segment elevation ≥ 1mm in two contiguous leads
  • : ≥ 2mm V2-V3
  • : ≥ 1.5mm V2-V3

Posterior STEMI

  • ST-segment depression V1-V3 Posterior ECG
  • ST-segment elevation ≥ 0.5mm in V7-V9

Sgarbossa Criteria

  • Evaluation for STEMI in LBBB or paced rhythm
  • Normal: ST-segment discordant with QRS

    • QRS associated with ST-segment depression
    • QRS associated with (commensurate) ST-segment elevation
  • Score ≥ 3 98% specific for MI

Elevation

  • Concordant ST-segment elevation ≥ 1mm in any lead (5 points)

Depression

  • Concordant ST-segment depression ≥ 1mm in V1-V3 (3 points)

Discordant Elevation

  • Discordant ST-segment elevation ≥ 5mm in any lead (2 points)

Modified Sgarbossa Criteria

  • ST:S ratio ≥ 0.25 in any lead
  • Presence of any criterion is positive

Other Causes of ST-segment Elevation

Benign Early Repolarization

  • Concave ST-segment elevation
  • Notch at J-point
  • Asymmetric T-waves (steeper descent)

Pericarditis

  • Diffuse ST-segment elevation (except aVR)
  • PR-segment depression
  • Ratio: ST-elevation to T-wave amplitude ≥ 0.25 in V6 suggests pericarditis

LVH Strain

  • ST-segment elevation in V1-V3 in the setting of LVH

LV Aneurysm

  • Q-waves with ST-segment elevation in precordial leads

Ischemia and Prior Infarcts

Wellens: Type A

Wellens: Type B

Q-waves

  • ≥ 40ms duration
  • Depth ≥ 25% of R-wave height

Syncope

ARVD

  • Epsilon wave

Brugada Syndrome: Type 1

  • Type 1: Coved ST-segment elevation

Brugada Syndrome: Type 2

  • Type 2: Saddle-back ST-segment elevation

HCM

  • Deep, narrow Q-waves

Wolff-Parkinson-White

  • Shortened PR-interval
  • Delta-wave

Other

Atrial Abnormalities

  1. Normal
  2. RAA: P-wave amplitude > 2.5mm in inferior leads
  3. LAA: P-wave duration increased (terminal negative portion >0.04s), amplitude of terminal negative component >1mm below isoelectric line in V1

Left Bundle Branch Block


  • QRS duration > 0.12s (3 boxes)
  • Broad or notched R-wave with prolonged upstroke in I, aVL, V5, V6
  • Associated ST-segment depression and T-wave inversion
  • Reciprocal changes in V1, V2 (deep S-wave)
  • Possible LAD

Right Bundle Branch Block


  • QRS duration > 0.12s (3 boxes)
  • RSR’ in V1, V2
  • Reciprocal changes in I, aVL, V5, V6 (deep S-wave)

Axes

All illustrations are available for free, licensed (along with all content on this site) under Creative Commons Attribution-ShareAlike 4.0 International Public License.

Downloads Page License

Sickle Cell Crises

Brief H&P

A 27 year-old male with sickle cell disease (HbSC) on hydroxurea and with a history of 2-3 hospitalizations per year for vaso-occlusive pain crises manifested by arthralgias and back pain presents to the emergency department with 3 days of worsening joint pain affecting his entire body but predominantly his knees and lower back. He is familiar with this pain and attempted therapy at home with ibuprofen, then hydrocodone-acetaminophen, and finally hydromorphone without improvement and presented to the emergency department.

On review of systems, he denied chest pain, cough, or shortness of breath. He has some periumbilical abdominal pain but tolerated normal oral intake on the day of presentation without vomiting nor changes in bowel habits. He otherwise denied fevers, peripheral numbness/weakness, urinary or fecal incontinence or retention. He similarly denies trauma, weight loss, night sweats, or intravenous drug use.

Objectively, the patient’s vital signs were normal and he was well-appearing. Mucous membranes were moist and skin turgor was normal. There were no appreciable joint effusions, warmth, nor limitation to active/passive range of motion of any joints. His back had no midline tenderness to palpation or percussion, normal range of motion in all axes and extremity sensation and strength testing were normal. Abdominal and genitourinary examinations were normal. The patient had preserved perineal sensation to light touch and normal rectal tone – a core temperature was obtained which was also normal.

Peripheral access was established and a parenteral dose of hydromorphone equivalent to his home oral dose was administered (0.015mg/kg). Repeat dosing was required at 15 minutes due to persistent pain scale of 10. Diphenhydramine and acetaminophen were also administered, for potential opioid-sparing effects, recognizing the limited evidence to support these relatively benign adjuncts.

Laboratory studies were notable for anemia (though stable compared to baseline measures), appropriate reticulocyte count, no evidence of hemolysis and with normal electrolytes and renal function.

A thorough history and examination did not identify a critical precipitant for the patient’s symptoms which were presumed to be secondary to a vaso-occlusive pain crisis. On reassessment, the patient’s pain was improved and an oral dose of hydromorphone was administered with continued observation and serial reassessments for two hours thereafter. The patient’s hematologist was available for follow-up the subsequent morning and the patient was discharged home.

Pharmacokinetics of Commonly-Used Opiate Analgesics1-3

Medication Route Onset Peak Duration
Morphine IV 5-10min 20min 3-5h
IM 15-30min 30-60min
PO 30min 1h
Oxycodone PO 10-15min 30-60min 3-6h
Hydrocodone PO 10-20min 4-8h
Fentanyl IV <1min 2-5min 30-60min
Hydromorphone IV 5min 10-20min 3-4h
PO 15-30min 30-60min
Codeine PO 30-60min 60-90min 4-6h

Spectrum of Sickle Cell Trait and Disease4

Algorithm for the Evaluation and Management of Sickle Cell Crises4-10

Algorithm for the Management of Sickle Cell Crises

References:

  1. Lexicomp Online®, Adult Drug Information, Hudson, Ohio: Lexi-Comp, Inc.; November 4, 2017.
  2. Trescot AM, Datta S, Lee M, Hansen H. Opioid pharmacology. Pain Physician. 2008;11(2 Suppl):S133-S153.
  3. Vieweg WVR, Lipps WFC, Fernandez A. Opioids and methadone equivalents for clinicians. Prim Care Companion J Clin Psychiatry. 2005;7(3):86-88.
  4. Glassberg J. Evidence-based management of sickle cell disease in the emergency department. Emergency Medicine Practice. 2011;13(8):1–20–quiz20.
  5. Raam R, Mallemat H, Jhun P, Herbert M. Sickle Cell Crisis and You: A How-to Guide. Ann Emerg Med. 2016;67(6):787-790. doi:10.1016/j.annemergmed.2016.04.016.
  6. Piel FB, Steinberg MH, Rees DC. Sickle Cell Disease. N Engl J Med. 2017;376(16):1561-1573. doi:10.1056/NEJMra1510865.
  7. Lovett PB, Sule HP, Lopez BL. Sickle cell disease in the emergency department. Emerg Med Clin North Am. 2014;32(3):629-647. doi:10.1016/j.emc.2014.04.011.
  8. Yawn BP, John-Sowah J. Management of Sickle Cell Disease: Recommendations From the 2014 Expert Panel Report. Vol 92. 2015:1069-1076.
  9. Zempsky WT. Evaluation and Treatment of Sickle Cell Pain in the Emergency Department: Paths to a Better Future. Clinical Pediatric Emergency Medicine. 2010;11(4):265-273. doi:10.1016/j.cpem.2010.09.002.
  10. Aliyu ZY, Tumblin AR, Kato GJ. Current therapy of sickle cell disease. Haematologica. 2006;91(1):7-10.

Principles of Neonatal Resuscitation

The following resource for neonatal resuscitation and neonatal critical care was developed with the guidance of Dr. Agrawal (Neonatology) while on rotation at the White Memorial Medical Center Neonatal Intensive Care Unit.

Endotracheal Tube Size1-3

Simplified Formula
Estimated gestational age in weeks ÷ 10 = round to nearest half-size uncuffed tube

NRP Recommendation

Gestation age (weeks) Weight (kg) ETT Size (ID, mm) Depth (cm from lip)
<28 <1.0 2.5 6-7
28-34 1.0-2.0 3.0 7-8
34-38 2.0-3.0 3.5 8-9
>38 >3.0 3.5-4.0 9-10

Laryngoscope Blade Size

Age Blade
Preterm 0
Term 1

Umbilical Vein Catheter Placement4

ED Indications
Unstable neonate
Contraindications
Omphalocele
Gastroschisis
Necrotizing enterocolitis
Depth
4-5cm or until blood return (for emergent placement)

Umbilical artery/vein catheter position on plain radiograph.

Umbilical catheter size

Weight (kg) Size (F)
<1.5 3.5
1.5-3.5 5
>3.5 8

Umbilical catheter positioning on plain radiographs

Umbilical venous catheter position can be verified with a plain radiograph. Positioning within the umbilical vein can be confirmed by tracing a cephalad trajectory from the insertion point at the umbilicus. An umbilical artery catheter will first pass caudally into the internal iliac artery before travelling cephalad into a common iliac artery and the abdominal aorta.

Medications5

Medication Dose
Epinephrine 0.1mL/kg (1:10,000) IV, 0.01mg/kg
Volume Expansion 10mL/kg (normal saline, blood)
Naloxone 0.1-0.2mg/kg
Dopamine 5-20mcg/kg/min IV infusion

Neonatal Physiology and Transition to Extrauterine Life6

An important principle in neonatal resuscitation is supporting the appropriate transition from intra- to extra-uterine life which is dependent on several key anatomic and physiologic changes occurring in an optimal environment.

Anatomy7

Fetal Circulation
Neonatal Circulation

Fetal Circulation

In the fetal circulatory system, oxygenated blood is delivered via the umbilical vein, entering the inferior vena cava via the ductus venosus. The majority of this oxygenated blood passes through the right atrium and into the left atrium through the foramen ovale to enter the systemic circulation.

Meanwhile, high pulmonary pulmonary vascular resistance (due to hypoxic vasoconstriction in fluid-filled alveoli) means that most of the deoxygenated right ventricular output is routed through the ductus arteriosus and enters into the systemic circulation – mixing with oxygenated blood distal to the highest priority end-organs (brain and heart), to be reoxygenated at the placenta.

Post-transition Circulation

The transition to extra-uterine life involves several key steps detailed below and is supported by appropriate ventilation, oxygenation and temperature regulation.

  1. Alveolar Fluid Clearance
    Catecholamine and hormone changes (predominantly corticosteroids) during the process of labor induce changes in enzymatic expression that result in the resorption of alveolar fluid into the interstitial space. At the time of delivery, negative intra-thoracic pressure from inspiration further promotes the resorption of alveolar fluid. Mechanical thoracic compression from delivery may also contribute.
  2. Respiration and Breathing
    Disconnection from the placenta ceases the transfer of placenta-derived factors including prostaglandins. The withdrawal of tonic inhibition of central respiratory drive from prostaglandins with cord clamping stimulates rhythmic breathing. The infant’s initial breaths and resultant lung expansion promotes alveolar expansion and stimulates surfactant production – this decreases alveolar surface tension, increases lung compliance and further facilitates breathing.
  3. Circulatory Changes
    At delivery, clamping the umbilical cord removes a large bed of low-resistance circulation, increasing systemic vascular resistance and systemic blood pressure. At the same time, lung expansion and alveolar aeration decreases pulmonary vascular resistance and pulmonary arterial pressures. At the ductus arteriosus, increased systemic vascular resistance combined with decreased pulmonary vascular resistance decreases shunting and contributes to closure. Similarly, as left atrial pressure approaches and exceeds right atrial pressure, right-to-left flow across the foramen ovale ceases. Collectively, these changes serve to effectively separate the left- and right-sided circulations.

NRP Resuscitation Algorithm5,8

Neonatal Resuscitation Algorithm

References

  1. Luten R, Kahn N, Wears R, Kissoon N. Predicting Endotracheal Tube Size by Length in Newborns. J Emerg Med. 2007;32(4):343-347. doi:10.1016/j.jemermed.2007.02.035.
  2. Peterson J, Johnson N, Deakins K, Wilson-Costello D, Jelovsek JE, Chatburn R. Accuracy of the 7-8-9 Rule for endotracheal tube placement in the neonate. J Perinatol. 2006;26(6):333-336. doi:10.1038/sj.jp.7211503.
  3. Kempley ST, Moreiras JW, Petrone FL. Endotracheal tube length for neonatal intubation. Resuscitation. 2008;77(3):369-373. doi:10.1016/j.resuscitation.2008.02.002.
  4. Anderson J, Leonard D, Braner DAV, Lai S, Tegtmeyer K. Videos in Clinical Medicine. Umbilical Vascular Catheterization. Vol 359. 2008:e18. doi:10.1056/NEJMvcm0800666.
  5. Association AAOPAAH. Textbook of Neonatal Resuscitation. 2016.
  6. Caraciolo J Fernandes MD. Physiologic transition from intrauterine to extrauterine life. UpToDate.
  7. Sadler TW. Langman’s Medical Embryology. Lippincott Williams & Wilkins; 2011.
  8. Perlman JM, Wyllie J, Kattwinkel J, et al. Part 7: Neonatal Resuscitation: 2015 International Consensus on Cardiopulmonary Resuscitation and Emergency Cardiovascular Care Science With Treatment Recommendations. In: Vol 132. American Heart Association, Inc.; 2015:S204-S241. doi:10.1161/CIR.0000000000000276.

Dysphagia

Brief H&P

A 47-year-old male with no known medical history presents with dysphagia. He reports 3 weeks of symptoms, describing difficulty predominantly with swallowing solid foods which is aided by the concomitant ingestion of liquids. He points to his throat as the area of discomfort, but has not noted any choking or coughing after attempts at swallowing. He occasionally suffers from “heartburn”, describing a burning sensation in his chest provoked by certain foods and was previously prescribed omeprazole which he has not taken for several years. He denies any prior surgeries, tobacco or alcohol use, relevant family history or similar symptoms in the past.

Physical examination was unrevealing, demonstrating a normal neurological examination, normal phonation, normal oropharynx and no appreciable neck masses. The patient was observed to comfortably swallow water.

He was discharged with gastroenterology follow-up and ultimately underwent esophagogastroduodenoscopy which demonstrated narrowing of the distal esophagus suggestive of a peptic stricture. Dilation was deferred in favor of resumption of proton pump inhibitor therapy.


Types of Dysphagia1,2

Oropharyngeal3
Characterized by difficulty initiating swallowing and accompanied by choking/coughing, nasopharyngeal regurgitation or aspiration.
Involved anatomy: Tongue, muscles of mastication, soft palate (elevation to close nasopharynx), suprahyoid muscles (elevate larynx), epiglottis (occlude airway), cricopharyngeus muscle (release upper esophageal sphincter). Neurological control predominantly coordinated by cranial nerves (V, VII, IX, X, XII)
Esophageal4
Delayed after initiating swallowing and characterized by a sensation of food bolus arresting in transit.
Involved anatomy: Skeletal and smooth muscle along the esophagus and lower esophageal sphincter. Neurological control predominantly coordinated by medulla

Important Historical Features5,6

  • Difficulty with liquids suggests motility problem
  • Difficulty with solids only or solids progressing to liquids suggests mechanical obstruction
  • Identify a history of head and neck surgery or radiation therapy
  • Identify a personal or family history of connective tissue disorder (scleroderma, RA, SLE) which may be associated with esophageal dysmotility
  • Review home medications (NSAID, bisphosphonate, potassium chloride, ferrous sulfate)
  • Immunocompromised patients are at risk for infectious esophagitis (Candida, CMV, HSV) which are generally associated with odynophagia
  • A history of heartburn may be associated with reflux-mediated complications such as erosive esophagitis, peptic stricture, and adenocarcinoma of the esophagus
  • Young patients are more likely to be affected by eosinophilic esophagitis
  • Patient localization of site of obstruction is generally accurate, patients are more accurate at localizing proximal than distal obstructions7

Algorithm for the Evaluation of Dysphagia8

Algorithm for the Evaluation of Dysphagia

Management9-11

Patients who are safely tolerating oral intake can be referred for outpatient gastroenterology evaluation. Admission should be considered for patients at high-risk for aspiration.

References

  1. Spieker MR. Evaluating dysphagia. Am Fam Physician. 2000;61(12):3639-3648.
  2. Abdel Jalil AA, Katzka DA, Castell DO. Approach to the patient with dysphagia. Am J Med. 2015;128(10):1138.e17-.e23. doi:10.1016/j.amjmed.2015.04.026.
  3. Shaker R. Oropharyngeal Dysphagia. Gastroenterol Hepatol (N Y). 2006;2(9):633-634.
  4. Galmiche JP, Clouse RE, Bálint A, et al. Functional esophageal disorders. Gastroenterology. 2006;130(5):1459-1465. doi:10.1053/j.gastro.2005.08.060.
  5. McCullough GH, Martino R. Clinical Evaluation of Patients with Dysphagia: Importance of History Taking and Physical Exam. In: Manual of Diagnostic and Therapeutic Techniques for Disorders of Deglutition. New York, NY: Springer New York; 2012:11-30. doi:10.1007/978-1-4614-3779-6_2.
  6. Cook IJ. Diagnostic evaluation of dysphagia. Nat Clin Pract Gastroenterol Hepatol. 2008;5(7):393-403. doi:10.1038/ncpgasthep1153.
  7. Wilcox CM, Alexander LN, Clark WS. Localization of an obstructing esophageal lesion. Is the patient accurate? Dig Dis Sci. 1995;40(10):2192-2196.
  8. Trate DM, Parkman HP, Fisher RS. Dysphagia. Evaluation, diagnosis, and treatment. Prim Care. 1996;23(3):417-432.
  9. American Gastroenterological Association medical position statement on management of oropharyngeal dysphagia. Gastroenterology. 1999;116(2):452-454. doi:10.1016/S0016-5085(99)70143-5.
  10. Spechler SJ. American Gastroenterological Association medical position statement on treatment of patients with dysphagia caused by benign disorders of the distal esophagus. Gastroenterology. 1999;117(1):229-232. doi:10.1016/S0016-5085(99)70572-X.
  11. Varadarajulu S, Eloubeidi MA, Patel RS, et al. The yield and the predictors of esophageal pathology when upper endoscopy is used for the initial evaluation of dysphagia. Gastrointest Endosc. 2005;61(7):804-808.

 

Bradycardia

Brief H&P:

A 38 year-old male with no medical history presents to the emergency department with abdominal pain. He had one episode each of non-bloody emesis followed by watery, non-bloody diarrhea and cited several sick contacts at home with similar symptoms. Vital signs were notable for bradycardia with a heart rate ranging from 38-46bpm though he was normotensive. The examination including abdominal examination was benign. A 12-lead electrocardiogram was obtained which demonstrated sinus bradycardia. The patient was asymptomatic during episodes of bradycardia and his heart rate responded appropriately during activity and on further history reported that he was an endurance athlete and runs multiple marathons each year. He was discharged after symptomatic improvement with anti-emetics.

Bradycardia 1

  • Definition: heart rate <60bpm
  • Sinus rhythm: upright P-wave in I, II, aV; inverted P-wave in aVR

Electrocardiographic Findings 1-4

  • Sinus bradycardia
    • Potentially asymptomatic and present in healthy individuals
  • Sinoatrial node dysfunction (sick sinus syndrome, SSS) 5,6
    • Sinus bradycardia
    • Sinus arrest
    • Tachy-brady syndrome (sinus bradycardia/arrest interspersed with SVT)
  • Atrioventricular block
    • 1st degree: PR prolongation, rarely symptomatic
    • 2nd degree: Intermittent interruption of conduction of atrial impulses to ventricles
      • Type 1: progressive PR prolongation leading to interrupted conduction
      • Type 2: fixed PR interval with interrupted conduction
    • 3rd degree: atrioventricular dissociation
  • Slow atrial fibrillation
    • Irregular RR interval without recognizable P-wave

Epidemiology7

  • Analysis of 277 patients presenting to the emergency department with “compromising” bradycardia.
  • Symptoms
    • Syncope (33%)
    • Dizziness (22%)
    • Angina (17%)
    • Dyspnea/Heart Failure (11%)
  • ECG
    • High-grade AV block (48%)
    • Sinus bradycardia (17%)
    • Sinus arrest (15%)
    • Slow atrial fibrillation (14%)
  • Cause
    • Primary (49%)
    • Drug (21%)
    • Ischemia/Infarction (14%)
    • Pacemaker failure (6%)
    • Intoxication (6%)
    • Electrolyte disorder (4%)

Important Historical Features8,9

  • Fever/travel
  • Chest pain
  • Cold intolerance, weight gain
  • Headache, AMS, trauma
  • Abdominal pain/distension
  • Medication changes

Important Examination Findings8,9

  • Perfusion (temperature, capillary refill)
  • Presence of fistula or hemodialysis catheter
  • Existing device (malfunction)

Workup8,9

  • ECG
  • Continuous telemetry monitoring
  • Labs
    • Potassium
    • Digoxin level
    • TFT
    • Infection titers (RPR, Lyme)
    • Cardiac enzymes

Management8,10

  • Unstable
    • Airway
    • Atropine 0.5mg IV q3-5min (maximum 3mg)
    • Dopamine/epinephrine infusion
    • Temporary pacemaker (transcutaneous, transvenous) with blood-pressure preserving sedation
    • Admission and evaluation for permanent pacemaker placement
  • Stable (outpatient evaluation)
    • Event monitor
    • Stress test (chronotropic incompetence)

Algorithm for the Evaluation and Management of Bradycardia

Algorithm for the evaluation and management of bradycardia

References

  1. Mangrum JM, DiMarco JP. The evaluation and management of bradycardia. N Engl J Med. 2000;342(10):703-709. doi:10.1056/NEJM200003093421006.
  2. Ufberg JW, Clark JS. Bradydysrhythmias and atrioventricular conduction blocks. Emergency Medicine Clinics of NA. 2006;24(1):1–9–v. doi:10.1016/j.emc.2005.08.006.
  3. Hayden GE, Brady WJ, Pollack M, Harrigan RA. Electrocardiographic manifestations: diagnosis of atrioventricular block in the Emergency Department. J Emerg Med. 2004;26(1):95-106. doi:10.1016/j.jemermed.2003.10.001.
  4. Da Costa D, Brady WJ, Edhouse J. Bradycardias and atrioventricular conduction block. BMJ. 2002;324(7336):535-538.
  5. Semelka M, Gera J, Usman S. Sick sinus syndrome: a review. Am Fam Physician. 2013;87(10):691-696.
  6. Ewy GA. Sick sinus syndrome: synopsis. J Am Coll Cardiol. 2014;64(6):539-540. doi:10.1016/j.jacc.2014.05.029.
  7. Sodeck GH, Domanovits H, Meron G, et al. Compromising bradycardia: management in the emergency department. Resuscitation. 2007;73(1):96-102. doi:10.1016/j.resuscitation.2006.08.006.
  8. Deal N. Evaluation and management of bradydysrhythmias in the emergency department. Emergency Medicine Practice. 2013;15(9):1–15–quiz15–6.
  9. Demla V, Rohra A. Emergency Department Evaluation and Management of Bradyarrhythmia. Hospital Medicine Clinics. 2015;4(4):526-539. doi:https://doi.org/10.1016/j.ehmc.2015.06.009.
  10. Brady WJ, Harrigan RA. Evaluation and management of bradyarrhythmias in the emergency department. Emergency Medicine Clinics of NA. 1998;16(2):361-388.

Hypotension

Brief H&P:

A 50 year-old male with a history of colonic mucinous adenocarcinoma on chemotherapy presented with a chief complaint of “vomiting”. He was unwilling to provide further history, repeating that he had vomited blood prior to presentation. His initial vital signs were notable for tachycardia. Physical examination showed some dried vomitus, brown in color, at the nares and lips; left upper quadrant abdominal tenderness to palpation; and guaiac-positive stool. Point-of-care hemoglobin was 3g/dL below the most recent measure two months prior. As his evaluation progressed, he developed hypotension and was transfused two units of uncrossmatched blood with adequate blood pressure response – he was started empirically on broad-spectrum antibiotics for an intra-abdominal source. Notable laboratory findings included a normal hemoglobin/hematocrit, acute kidney injury, and elevated anion gap metabolic acidosis presumably attributable to serum lactate of 10.7mmol/L. Computed tomography of the abdomen and pelvis demonstrated pneumoperitoneum with complex ascites concerning for bowel perforation. The patient deteriorated, was intubated, started on vasopressors and admitted to the surgical intensive care unit. The initial operative report noted extensive adhesions and perforated small bowel with feculent peritonitis. He has since undergone multiple further abdominal surgeries and remains critically ill.

Imaging

IM-0001-0032
IM-0001-0033
IM-0001-0034
IM-0001-0035
IM-0001-0036
IM-0001-0037
IM-0001-0038
IM-0001-0039
IM-0001-0040
IM-0001-0041
IM-0001-0042
IM-0001-0043
IM-0001-0044
IM-0001-0045
IM-0001-0046
IM-0001-0047
IM-0001-0048
IM-0001-0049
IM-0001-0050

CT Abdomen/Pelvis

Free air is seen diffusely in the non-dependent portions of the abdomen: in the anterior abdomen and pelvis, inferior to the diaphragm, and in the perisplenic region. There is complex free fluid in the abdomen.

Algorithm for the Evaluation of Hypotension1

This process for the evaluation of hypotension in the emergency department was developed by Dr. Ravi Morchi. In the case above, a systematic approach to the evaluation of hypotension using ultrasonography and appropriately detailed physical examination may have expedited the patient’s care. The expertly-designed algorithm traverses the cardiovascular system, halting at evaluable checkpoints that may contribute to hypotension.

  1. The process begins with the cardiac conduction system to identify malignant dysrhythmias (bradycardia, or non-sinus tachycardia >170bpm), which, in unstable patients are managed with electricity.
  2. The next step assesses intravascular volume with physical examination or bedside ultrasonography of the inferior vena cava. Decreased right atrial pressure (whether due to hypovolemia, hemorrhage, or a distributive process) is evidenced by a small and collapsible IVC. If hemorrhage is suspected, further ultrasonography with FAST and evaluation of the abdominal aorta may identify intra- or retroperitoneal bleeding.
  3. If a normal or elevated right atrial pressure is identified, evaluate for dissociation between the RAP and left ventricular end-diastolic volume. This is typically caused by a pre- or intra-pulmonary obstructive process such as tension pneumothorax, cardiac tamponade, massive pulmonary embolism, pulmonary hypertension, or elevated intra-thoracic pressures secondary to air-trapping. Thoracic ultrasonography can identify pneumothorax, pericardial effusion, or signs of elevated right ventricular systolic pressures (RV:LV, septal flattening).
  4. Assuming adequate intra-vascular volume is arriving at the left ventricle, rapid echocardiography can be used to provide a gross estimate of cardiac contractility and point to a cardiogenic process. If there is no obvious pump failure, auscultation may reveal murmurs that would suggest systolic output is refluxing to lower-resistance routes (ex. mitral insufficiency, aortic insufficiency, or ventricular septal defect).
  5. Finally, if the heart rate is suitable, volume deficits are not grossly at fault, no obstructive process is suspected, and cardiac contractility is adequate and directed appropriately through the vascular tree, the cause may be distributive. Physical examination may reveal dilated capillary beds and low systemic vascular resistance.

Algorithm for the Evaluation of Hypotension

Guided Lecture

EM Ed
Watch “The Transiently Hypotensive Patient: Who Cares?” from EM Ed. In this lecture Dr. Basrai reviews the diagnostic pathway for a patient who presents with transient hypotension.

References

  1. Morchi R. Diagnosis Deconstructed: Solving Hypotension in 30 Seconds. Emergency Medicine News. 2015.

Wellens Syndrome

Case Presentation

49M with a history of hypertension who presented to his primary physician for routine follow-up and was referred to the ED for an abnormal ECG. He denied chest pain, shortness of breath, or any limitation to baseline exercise tolerance. His vital signs were notable for systolic hypertension and his examination was unremarkable. A chest x-ray showed no acute cardiopulmonary findings. His initial ECG demonstrated a biphasic T-wave in V2 and deep, symmetric T-wave inversions in V3-V6. His initial serum troponin was markedly elevated at 3.499. He was admitted and urgent coronary angiography revealed proximal LAD stenosis (70%), mid-LAD stenosis (85%) and 1st right posterolateral stenosis (85%) which were stented. He was discharged on post-procedure day one and has remained asymptomatic at outpatient follow-up.

Presentation ECG
Presentation ECG

Presentation ECG

Biphasic T-wave in V2, deep and symmetric T-wave inversions in V3-V4

Post-Catheterization ECG
Post-Catheterization ECG

Post-Catheterization ECG

Resolution of biphasic T-wave and T-wave inversions

History1

Initially described in 1982 where a subset of patients who did poorly with medical management of “impending myocardial infarction” (essentialy unstable angina) were found to have characteristic ECG changes. These patients were noted to be at increased risk for extensive anterior wall myocardial infarctions due to proximal LAD stenosis.

Wellens ECG patterns

Criteria2,3

  1. History of chest pain
  2. Normal or slightly-elevated cardiac enzymes
  3. No precordial Q-waves
  4. Isoelectric or <1mm ST-segment elevation
  5. Pattern present in pain-free state
  6. Type A (25%): Biphasic T-wave in V2/V3
  7. Type B (75%): Deep, symmetrically inverted T-waves in V2/V3

Clinical Significance3

Wellens Syndrome (or LAD coronary T-wave syndrome) represents a “pre-infarction” stage of coronary artery disease manifested by critical LAD stenosis. The natural history includes progression to extensive anterior wall myocardial infarction, often associated with severe left ventricular systolic dysfunction, cardiogenic shock and death. These changes may be mistaken for “non-specific” T-wave changes (which in the presence of a non-concerning history and typically non-elevated cardiac markers) may lead providers to inappropriate dispositions such a stress testing which is contraindicated. Recognition of this pattern and its appropriate management (urgent coronary angiography) is critical.

Case Summary

The case presented above is atypical. The patient had no history of chest pain and cardiac enzymes were significantly elevated – two features which are uncommon in Wellens Syndrome. However, the patient’s elevated cardiac biomarkers led to admission and angiography with identification of the characteristic proximal LAD stenosis (and other disease).

References:

  1. de Zwaan C, Bär FW, Wellens HJ. Characteristic electrocardiographic pattern indicating a critical stenosis high in left anterior descending coronary artery in patients admitted because of impending myocardial infarction. Am Heart J. 1982;103(4 Pt 2):730-736.
  2. Tandy TK, Bottomy DP, Lewis JG. Wellens’ syndrome. YMEM. 1999;33(3):347-351.
  3. Rhinehardt J, Brady WJ, Perron AD, Mattu A. Electrocardiographic manifestations of Wellens’ syndrome. American Journal of Emergency Medicine. 2002;20(7):638-643. doi:10.1053/ajem.2002.34800.
  4. Mead N, O Keefe K. Wellen′s Syndrome: An Ominous EKG pattern. J Emerg Trauma Shock. 2009;2(3):206– doi:10.4103/0974-2700.55347.
  5. Kannan L, Figueredo VM. Images in clinical medicine. Wellens’ syndrome. N Engl J Med. 2015;372(1):66. doi:10.1056/NEJMicm1400946.

Hypocalcemia

Brief H&P:

34M with a history of HTN, polysubstance abuse, presenting with muscle cramps. He reported onset of diffuse muscle cramping 1-hour prior to presentation while showering. Symptoms involved bilateral upper and lower extremities and resolved spontaneously.

On initial evaluation, the patient was tachycardic and hypertensive. Examination was notable for tremors in bilateral upper extremities with outstretched hands, as well as of extended tongue. Other notable findings included spasm of the upper extremity during blood pressure measurement, hyperreflexia and clonus.

Laboratory evaluation was notable for normal total calcium level, low ionized calcium level, primary respiratory alkalosis, and elevated anion gap metabolic acidosis.

The patient was treated with intravenous fluids, benzodiazepines for alcohol withdrawal, and calcium gluconate 4g IV and was admitted.

Calcium Homeostasis1

  • Fraction
    • 15% bound to anions (phosphate, lactate, citrate)
    • 40% bound to albumin
    • 45% free (regulated by PTH, Vit-D)
  • Conditions causing changes in total calcium (without affecting ionized calcium)
    • Low albumin causes hypocalcemia. Corrected = measured + [0.8 x (4-albumin)]
    • Elevated albumin causes hypercalcemia
    • Multiple myeloma causes hypercalcemia
  • Conditions causing changes in ionized calcium (without affecting total calcium)
    • Alkalemia causes increased ionized calcium binding to albumin and decreases ionized calcium levels
    • Hyperphosphatemia causes increased ionized calcium binding to phosphate and decreases ionized calcium levels
    • Hyperparathyroidism causes decreased ionized calcium binding to albumin and increases ionized calcium levels

Causes of Hypocalcemia1,2,3

Algorithm for the Evaluation of Hypocalcemia

Symptoms1

Acute Chronic

Neuromuscular

  • Paresthesia
  • Tetany
  • Carpopedal spasm
  • Trousseau
  • Chvostek
  • Seizure
  • Laryngospasm

Cardiac

  • QT prolongation
  • Hypotension
  • Heart failure
  • Arrhythmia

CNS

  • Basal ganglia calcifications
  • EPS
  • Parkinsonism
  • Dementia

Ophthalmologic

  • Cataracts

Management

  • Severe (symptomatic, QT prolongation)
    • Calcium gluconate 1-2g IV in 50mL of D5W over 10-20min followed by slow infusion of additional 2g over 2 hours.
  • Asymptomatic
    • Calcium gluconate 1g PO q6h
    • Calcitriol 0.2mcg PO BID

References:

  1. Yu, AS. Relation between total and ionized serum calcium concentrations. In: UpToDate, Post TW (Ed), UpToDate, Waltham, MA. (Accessed on October 6th, 2016.)
  2. Cooper MS, Gittoes NJL. Diagnosis and management of hypocalcaemia. BMJ. 2008;336(7656):1298-1302. doi:10.1136/bmj.39582.589433.BE.
  3. Hannan FM, Thakker RV. Investigating hypocalcaemia. BMJ. 2013;346(may09 1):f2213-f2213. doi:10.1136/bmj.f2213.

Neurosyphilis

Brief H&P

A young male with a history of HIV (untreated for the last year, with unknown CD4 count), and syphilis (reportedly treated with an intramuscular injection 1 year ago), presents with 4 months of a painful rash on the palms and soles and diplopia. Examination revealed the rash pictured below, ocular examination with minimal papilledema and anterior chamber inflammation.

Labs were unremarkable. CSF sampling was notable for 34 WBC’s with lymphocyte predominance (92%), and elevated protein (56mg/dL). The patient was admitted for syphilis with presumed neurosyphilis. Serum RPR titer was elevated at 1:64,  FTA-ABS and CSF VDRL were reactive. The patient was treated with intravenous penicillin and anti-retroviral therapy was reinitiated.

Epidemiology1

  • Transmission
    • Sexual contact (estimated transmission probability 60% per partner)
    • Trans-placental
  • Race/Sex
    •  African-American, Hispanic
    • Male > Female
    • Male (primary syphilis), female (secondary syphilis) – lesion visibility
    • Urban > rural

Natural History1

Stage Signs/Symptoms Incubation Period
Primary Chancre, reginal lymphadenopathy 3 weeks
Secondary Rash, fever, malaise, generalized lymphadenopathy, mucous membrane lesions, condyloma lata, headache, meningitis 2-12 weeks
Latent Asymptomatic Early (<1 year)

Late (>1 year)

Tertiary Cardiovascular:

Aortic aneurysm, aortic insufficiency, coronary artery ostial stenosis

<2 years
CNS:
Acute syphilitic meningitis: headache, confusion, meningeal irritation <2 years
Meningovascular: cranial nerve palsy 5-7 years
General paresis: headache, vertigo, personality changes, vascular event 5-7 years
Tabes dorsalis: dementia, ataxia, Argyl-Robertson, [arrow-down] proprioception 10-20 years
Gumma:

Local tissue destruction

1-46 years

Diagnosis1

  • Serologic
    • Non-treponemal (screening)
      • RPR, VDRL
      • Limitations:  sensitivity, false positive (age, pregnancy, drugs, malignancy, autoimmune, viral infections)
    • Treponemal (confirmatory)
      • FTA-ABS
    • Neurosyphilis
      • Indications for CSF sampling: neurologic/ophthalmologic symptoms, tertiary syphilis (aortitis, gumma, iritis), HIV coinfection with elevated RPR titer (> 1:32)
      • CSF: leukocytosis (predominantly lymphocytes),  protein
      • CSF VDRL reactive
      • Negative CSF FTA-ABS may rule out neurosyphilis

Syphilis in HIV-infected Individuals2

  • Primary: larger and more lesion, multiple ulcers
  • Secondary: genital ulcers more common, higher RPR/VDRL titers
  • Tertiary: possibly more rapid progression to neurosyphilis

References

  1. Singh AE, Romanowski B. Syphilis: review with emphasis on clinical, epidemiologic, and some biologic features. Clin Microbiol Rev. 1999;12(2):187-209.
  2. French P. Syphilis. BMJ. 2007;334(7585):143-147. doi:10.1136/bmj.39085.518148.BE.